download scanned papers here - Paper 1    Paper II


UPSC PRELIMS 2022 - PAPER I - ANSWER KEY

Exam Analysis HOME       2022 - Paper I - English      Paper II - E       Paper I - Hindi       Paper II - H      



BIRD'S EYEVIEW

Exam Analysis HOME       2022 - Paper I - English      Paper II - E       Paper I - Hindi       Paper II - H      

  • The Union Public Service Commission (UPSC) Prelims 2022 Paper 1 and 2 was conducted by the Commission at various centers across the country.
  • Exam Date: June 5, 2022 (Sunday) | Time: 09:30 AM to 11:30 AM (Two Hours) | Nature of the Exam: MCQ pen and paper test | Total number of Questions: 100 | Total Marks: 200 marks paper with 33% negative marking for every incorrect answer.
  • New Variety of Questions: A new variety of questions was introduced this year into the GS paper, where the question asked the candidates to choose the right number of matching options. Some questions o “International Relations” were asked on this patten. The new question pattern was tricky and time-consuming. There was total 7 questions of this type. This would impact the cut-off.
  • Comeback: Medieval History terminologies and questions on International Relations were a comeback. A greater number of questions were asked on international regions, organizations and conventions. Questions about Afghanistan, Central Asia, East Europe & Africa were asked.
  • Linkage of Current Affairs: Questions needed interlinking of knowledge between the Current Affairs and the Static knowledge. But it was not too tough to crack.
  • Balanced difficulty level: This year’s question paper covered all sections mentioned in the UPSC syllabus with good weightage to most. The paper was generally uniform. Nearly equal weightage given to all major topics. Balanced distribution. But this is no guarantee of what happens in 2023!
  • Toughness: At the first glance, the paper looks moderate since the topics look familiar but the devil lies in the details! Superficial knowledge wasn’t enough; the intricacies made the difference this time.
  • Conceptual clarity: It was of paramount importance. Several questions were tricky - where clarity of concepts was of paramount importance. Knowing a concept definition wasn’t enough, several were application-based questions, especially in economy.
  • Mapping: It got a lion’s share of questions this time (in comparison to previous years). Most were in the news- thus reading the newspaper along with an atlas is a good habit to cultivate now. Domestic (wetlands, rivers) and international regions (Levant, Afghanistan border, etc) were equally important.
  • Subject wise Analysis:
    1. Environment and Ecology saw some good questions, with many micro details being asked. A total of 15 questions appeared (last year 21)
    2. Science and Technology questions were 12 in number (last year 4). The focus was on IT and communications technology
    3. Polity questions were factual but doable and were relatively easier than in previous years. A total of 10 were asked (last year 17)
    4. Economy and banking questions were related to current events and demanded deep understanding of concepts even for guesswork. Superficial knowledge would not help.
    5. Ancient and Medieval history, and Art and Culture questions were tricky – needed lots of memorization of details and terminologies. Focus area were the Buddhism, Jainism, Temples, literature and texts, Mongols, Early British Expansion, Constitutional History, Revolutionaries, Sangam Literature, the Mauryan Dynasty. Some of the questions can be termed difficult to answer.
    6. Two Government Schemes were asked.
    7. Geography questions were less conceptual, and a mix of factual and location-based questions were asked.
  • Strategy: The paper was prepared smartly: precluded the usage of elimination strategies in some questions, and required one to actually have sound knowledge about all the answers. Interestingly, the scope of using elimination techniques was removed due to a new pattern of options. An aspirant has to put in consistent efforts and has to do multiple revisions.
  • Expected Cut-offs: Considering the level of the question paper, the cut-off is predicted to be in the range of 90-95; higher than last year's cut off (87.54 for Gen category).
TOPICWISE DISTRIBUTION

Exam Analysis HOME       2022 - Paper I - English      Paper II - E       Paper I - Hindi       Paper II - H      






DETAILED QUESTIONWISE SOLUTIONS (Set B)


Exam Analysis HOME       2022 - Paper I - English      Paper II - E       Paper I - Hindi       Paper II - H      


1. "Climate Action Tracker" which monitors the emission reduction pledges of different countries is a:

  1. Database created by coalition of research organisations
  2. Wing of "International Panel of Climate Change"
  3. Committee under "United Nations Framework Convention on Climate Change"
  4. Agency promoted and financed by United Nations Environment Programme and World Bank

Sol. Ans.(a). The Climate Action Tracker is an independent scientific analysis that tracks government climate action and measures it against the globally agreed Paris Agreement aim of “to hold warming well below 2°C, and pursuing efforts to limit warming to 1.5°C.” It is a collaboration of two organisations - Climate Analytics and NewClimate Institute. It has been providing independent analysis since 2009. It is not related to IPCC, UNFCC, UNEP and WB. So, option (B), (C) and (D) are not true

( This question was directly covered in PT’s Powerpack Course – enquire and enroll here - https://bit.ly/upscias )

Source: https://climateactiontracker.org/about/


2. Consider the following statements:

  1. "The Climate Group" is an international non-profit organization that drives climate action by building large networks and runs them.
  2. The International Energy Agency in partnership with the Climate Group launched a global initiative "EPIOO".
  3. EP100 brings together leading companies committed to driving innovation in energy efficiency and increasing competitiveness while delivering on emission reduction goals.
  4. Some Indian companies are members of EP100.
  5. The International Energy Agency is the Secretariat to the "Under2 Coalition".

Which of the statements given above are correct?

  1. 1, 2, 4 and 5
  2. 1, 3 and 4 only
  3. 2, 3 and 5 only
  4. 1, 2, 3, 4 and 5

Sol. Ans.(b). “The Climate Group” is the Secretariat to the Under2 Coalition and not the International Energy Agency as given in statement 5. So statement 5 should not be part of correct answer. Such option is only option (b).

Source:
https://www.theclimategroup.org/frequently-asked-questions#:~:text=The%20Climate%20Group%20is%20the,holding%20the%20governments%20to%20account.


3. "If rainforests and tropical forests are the lungs of the Earth, then surely wetlands function as its kidneys." Which one of the following functions of wetlands best reflects the above statement?

  1. The water cycle in wetlands involves ' surface runoff, subsoil percolation and evaporation.
  2. Algae form the nutrient base upon which fish, crustaceans, molluscs, birds, reptiles and mammals thrive.
  3. Wetlands play a vital role in maintaining sedimentation balance and soil stabilization.
  4. Aquatic plants absorb heavy metals and excess nutrients.

Sol. Ans.(d). While tropical rainforests are often called the lungs of the Earth, wetlands are described as the kidneys. They are described so as they can absorb large quantities of water, filtering pollutants and helping to prevent floods. Aquatic plants are sinks for heavy metals in aquatic ecosystems, and that different aquatic plant species accumulate variable amounts of different heavy metals. For example, studies have shown that floating plants such as water hyacinth (Eichhornia crassipes), emergent plants such as cattail (Typha. latifolia), and submerged plants such as Hydrilla verticillata, Ceratophyllum demersum, and Potamogeton malaianus have significant capacities to accumulate heavy metals. They have also become key allies in the fight against climate change, as they can capture CO2 from the atmosphere. Wetlands are still disappearing three times faster than forests. (Option (c) is also pretty close)

( This question was directly covered in PT’s Powerpack Course – enquire and enroll here - https://bit.ly/upscias )

Source: https://www.ncbi.nlm.nih.gov/pmc/articles/PMC6313464/


4. In the context of WHO Air Quality Guidelines, consider the following statements:

  1. The 24-hour mean of PM2.5 should not exceed 15 µg/m3 and annual mean of PM2.5 should not exceed 5 fig/m3.
  2. In a year, the highest levels of ozone pollution occur during the periods of inclement weather.
  3. PM10 can penetrate the lung barrier and enter the bloodstream.
  4. Excessive ozone in the air can trigger asthma.

Which of the statements given above are correct?

  1. 1, 3 and 4
  2. 1 and 4 only
  3. 2, 3 and 4
  4. 1 and 2 only

Sol. Ans.(b). Check Statement 3 – PM 2.5 can penetrate the lung barrier and enter the bloodstream but not PM 10. So, 3 should not be part of correct answer option. Hence (a) and (c) are ruled out. Out of 2 and 4, it is true that excessive ozone can trigger asthma, whereas proofs for highest level of ozone pollution during inclement weather is not evident. Statement 1 is correct (Please refer to the source given below).

( This question was directly covered in PT’s Powerpack Course – enquire and enroll here - https://bit.ly/upscias )

Source 1: https://www.breeze-technologies.de/blog/new-2021-who-air-quality-guideline-limits/

Source 2:
https://www.epa.gov/ground-level-ozone-pollution/health-effects-ozone-pollution#:~:text=Long%2Dterm%20exposure%20to%20ozone,with%20deaths%20from%20respiratory%20causes.


5. With reference to "Gucchi" sometimes mentioned in the news, consider the following statements:

  1. It is a fungus.
  2. It grows in some Himalayan forest areas.
  3. It is commercially cultivated in the Himalayan foothills of north-eastern India.

Which of the statements given above is/are correct?

  1. only
  2. 3 only
  3. 1 and 2
  4. 2 and 3

Sol. Ans.(c). Gucchi is known to be one of the most expensive mushrooms, with only 500 gm of it costing up to Rs 18000. What is a mushroom? A mushroom or toadstool is the fleshy, spore-bearing fruiting body of a fungus, typically produced above ground, on soil, or on its food source. So 1 is correct and options (b) and (c) are ruled out. 2 is also correct as Guchchi mushrooms are grown in the Himalayan region after the snowfall • They have a unique flavour and earthy aroma (refer source for more information).

Source : https://economictimes.indiatimes.com/gucchi-wild-mushrooms-from-himalayas-worth-their-weight-in-gold/articleshow/19007096.cms?from=mdr


6. With reference to polyethylene terephthalate, the use of which is so widespread in our daily lives, consider the following statements:

  1. Its fibres can be blended with wool and cotton fibres to reinforce their properties.
  2. Containers made of it can be used to store any alcoholic beverage.
  3. Bottles made of it can be recycled into other products.
  4. Articles made of it can be easily disposed of by incineration without causing greenhouse gas emissions.

Which of the statements given above are correct?

  1. 1 and 3
  2. 2 and 4
  3. 1 and 4
  4. 2 and 3

Sol. Ans.(a). PET (or PETE) is a thermoplastic, not a thermoset plastic. So it cannot be incinerated but will turn into liquid instead. So statement 4 is wrong, and (b) and (c) are ruled out. Also, PET is made into a high-strength textile fibre marketed by some firms. The stiffness of PET fibres makes them highly resistant to deformation, so they impart excellent resistance to wrinkling in fabrics. They are often used in durable-press blends with other fibres such as rayon, wool, and cotton, reinforcing the inherent properties of those fibres while contributing to the ability of the fabric to recover from wrinkling. So 1 is correct, options (b) and (d) are ruled out. PET is the most widely recycled plastic. PET bottles and containers are commonly melted down and spun into fibres for fibrefill or carpets. So 3 is correct.

Source 1 : https://www.britannica.com/science/polyethylene-terephthalate#ref1049366

Source 2 : https://www.thehindu.com/news/cities/mumbai/news/Govt-rules-out-plastic-liquor-bottles/article13994794.ece


7. Which of the following is not a bird?

  1. Golden Mahseer
  2. Indian Nightjar
  3. Spoonbill
  4. White Ibis

Which of the statements given above are correct?

  1. 1 and 3
  2. 2 and 4
  3. 1 and 4
  4. 2 and 3

Sol. Ans.(a). Easy question. Golden Mahseer is a fish, which is labelled “endangered”. Other three are bird species.

Source : https://www.wwfindia.org/about_wwf/priority_species/threatened_species/golden_mahseer/


8. Which of the following are nitrogen-fixing plants?

  1. Alfalfa
  2. Amaranth
  3. Chickpea
  4. Clover
  5. Purslane (Kulfa)
  6. Spinach

Select the correct answer using the code given below:

  1. 1, 3 and 4 only
  2. 1, 3, 5 and 6 only
  3. 2, 4, 5 and 6 only
  4. 1, 2, 4, 5 and 6

Sol. Ans.(a). This is a question from agriculture. Plants that contribute to nitrogen fixation include the legume family – Fabaceae – with taxa such as clover, soybeans, alfalfa, lupins, peanuts, and rooibos. They contain symbiotic bacteria called Rhizobia within nodules in their root systems, producing nitrogen compounds that help the plant to grow and compete with other plants. When the plant dies, the fixed nitrogen is released, making it available to other plants and this helps to fertilize the soil. The great majority of legumes have this association, but a few genera (e.g., Styphnolobium) do not. In many traditional and organic farming practices, fields are rotated through various types of crops, which usually includes one consisting mainly or entirely of clover or buckwheat (family Polygonaceae), which were often referred to as "green manure.".

Source :
https://en.wikipedia.org/wiki/Category:Nitrogen-fixing_crops#:~:text=Plants%20that%20contribute%20to%20nitrogen,lupins%2C%20peanuts%2C%20and%20rooibos.


9. "Biorock technology" is talked about in which one of the following situations?

  1. Restoration of damaged coral reefs
  2. Development of building materials using plant residues
  3. Identification of areas for exploration/extraction of shale gas
  4. Providing salt licks for wild animals in forests/protected areas

Sol. Ans.(a). The “Biorock” technology is an innovative process originally invented in 1976 by Wolf Hilbertz to produce natural building materials in the sea. Biorock materials are the only marine construction material that grow, get stronger with age, and are self-repairing. The technology was called Biorock, because it not only grew hard limestone rock for structural purposes, it greatly increased growth of corals and all marine organisms. Coral reefs are in danger due to global warming, and hence this technology is considered very useful.

( This question was directly covered in PT’s Powerpack Course – enquire and enroll here - https://bit.ly/upscias )

Source : https://www.globalcoral.org/biorock-coral-reef-marine-habitat-restoration/


10. The "Miyawaki method" is well known for the:

  1. Promotion of commercial farming in arid and semi-arid areas
  2. Development of gardens using genetically modified flora
  3. Creation of mini forests in urban areas
  4. Harvesting wind energy on coastal areas and on sea surfaces

Sol. Ans.(c). “Miyawaki” is a technique pioneered by Japanese botanist Akira Miyawaki, that helps build dense, native forests, and can be very useful in mitigating the worst effects of urban heat islands. The approach is supposed to ensure that plant growth is 10 times faster and the resulting plantation is 30 times denser than usual. So, (c) is the correct answer.

( This question was directly covered in PT’s Powerpack Course – enquire and enroll here - https://bit.ly/upscias )

Source :
https://www.sei.org/about-sei/press-room/how-the-miyawaki-method-can-transform-indian-cities/#:~:text=The%20method%20involves%20planting%20two,insects%2C%20and%20create%20carbon%20sinks.


11. Consider the following:

  1. Aarogya Setu
  2. CoWIN
  3. DigiLocker
  4. DIKSHA

Which of the above are built on top of open-source digital platforms?

  1. 1 and 2 only
  2. 2, 3 and 4 only
  3. 1, 3 and 4 only
  4. 1, 2, 3 and 4

Sol. Ans.(d). All other Apps CoWIN, DigiLocker and DIKSHA are built on top of open-source digital platform. That is called the FOSS – Free and Open Source Software, that the government of India has used for many publicly used applications. Aarogya Setu was originally not built that way. It is an Indian COVID–19 "contact tracing, syndromic mapping and self-assessment" digital service, primarily a mobile app, developed by the National Informatics Centre under the Ministry of Electronics and Information Technology. It was made open source later, and the PM announced in July 2021 that it was available to all countries that wanted it. But the question is framed in such a way that marking (b) will be risky, so go with (d). Yes, there’s some confusion in this one!

Source : https://www.thehindu.com/sci-tech/technology/aarogya-setu-app-is-now-open-source-what-does-it-mean/article31689459.ece


12. With reference to Web 3-0, consider the following statements:

  1. Web 3-0 technology enables people to control their own data.
  2. In Web 3-0 world, there can be block chain based social networks.
  3. Web 3-0 is operated by users collectively rather than a corporation.

Which of the statements given above are correct?

  1. 1 and 2 only
  2. 2 and 3 only
  3. 1 and 3 only
  4. 1, 2 and 3

Sol. Ans.(d). Web 3.0 represents the next iteration or phase of the evolution of the web/internet and may prove as disruptive as Web 2.0 was. Web 3.0 is built upon the core concepts of decentralization, openness, and greater user utility.

Decentralization: “No permission is needed from a central authority to post anything on the web, there is no central controlling node, and so no single point of failure...and no ‘kill switch’! This also implies freedom from indiscriminate censorship and surveillance.” This is a core tenet of Web 3.0. In Web 2.0, computers use HTTP in the form of unique web addresses to find information, which is stored at a fixed location, generally on a single server. With Web 3.0, because information would be found based on its content, it could be stored in multiple locations simultaneously and hence be decentralized. This would break down the massive databases currently held by internet giants like Meta and Google and would hand greater control to users. Hence 3 is true.

Bottom-up design: “Instead of code being written and controlled by a small group of experts, it was developed in full view of everyone, encouraging maximum participation and experimentation.”

With Web 3.0, the data generated by disparate and increasingly powerful computing resources, including mobile phones, desktops, appliances, vehicles, and sensors, will be sold by users through decentralized data networks, ensuring that users retain ownership control. Hence 1 is true.

Trustless and permissionless: In addition to decentralization and being based upon open-source software, Web 3.0 will also be trustless (i.e., the network will allow participants to interact directly without going through a trusted intermediary) and permissionless (meaning that anyone can participate without authorization from a governing body). As a result, Web 3.0 applications will run on blockchains or decentralized peer-to-peer networks, or a combination thereof—such decentralized apps are referred to as dApps.

( This question was directly covered in PT’s Powerpack Course – enquire and enroll here - https://bit.ly/upscias )

Source : https://www.investopedia.com/web-20-web-30-5208698


13. With reference to "Software as a Service (SaaS)", consider the following statements:

  1. SaaS buyers can customise the user interface and can change data fields.
  2. SaaS users can access their data through their mobile devices.
  3. Outlook, Hotmail and Yahoo! Mail are forms of SaaS.

Which of the statements given above are correct?

  1. 1 and 2 only
  2. 2 and 3 only
  3. 1 and 3 only
  4. 1, 2 and 3

Sol. Ans.(d). SaaS applications are often customizable and can be integrated with other business applications, especially across applications from a common software provider obviously we can use them on mobile phones. Outlook, Hotmail or Yahoo! Mail are based on SaaS forms.

Refer to the source - https://azure.microsoft.com/en-in/overview/what-is-saas/

( This question was directly covered in PT’s Powerpack Course – enquire and enroll here - https://bit.ly/upscias )


14. Which one of the following statements best reflects the idea behind the "Fractional Orbital Bombardment System" often talked about in media?

  1. A hypersonic missile is launched into space to counter the asteroid approaching the Earth and explode it in space.
  2. A spacecraft lands on another planet after making several orbital motions.
  3. A missile is put into a stable orbit around the Earth and deorbits over a target on the Earth.
  4. A spacecraft moves along a comet with the same speed and places a probe on its surface.

Sol. Ans.(c). A FOBS (Fractional Orbital Bombardment System) is a warhead delivery system that uses a low earth orbit towards its target destination. Just before reaching the target, it deorbits through a retrograde engine burn. It was once considered a very dangerous system as detecting such missiles would be extremely difficult. It could travel overhead the south pole before hitting its target, for example.

Source - https://en.wikipedia.org/wiki/Fractional_Orbital_Bombardment_System


15. Which one of the following is the context in which the term "qubit" is mentioned?

  1. Cloud Services
  2. Quantum Computing
  3. Visible Light Communication Technologies
  4. Wireless Communication Technologies

Sol. Ans.(b). This was a very easy question (a sitter). In quantum computing, a qubit or quantum bit is a basic unit of quantum information—the quantum version of the classic binary bit physically realized with a two-state device.

( This question was directly covered in PT’s Powerpack Course – enquire and enroll here - https://bit.ly/upscias )

Source - https://en.wikipedia.org/wiki/Qubit


16. Consider the following communication technologies:

  1. Closed-circuit Television
  2. Radio Frequency Identification
  3. Wireless Local Area Network

Which of the above are considered Short-Range devices/technologies?

  1. 1 and 2 only
  2. 2 and 3 only
  3. 1 and 3 only
  4. 1, 2 and 3

Sol. Ans.(d). All are short-range devices.

CCTV (closed-circuit television) is a TV system in which signals are not publicly distributed but are monitored, primarily for surveillance and security purposes. CCTV relies on strategic placement of cameras and private observation of the camera's input on monitors.

Radio Frequency Identification (RFID) is a technology that uses radio waves to passively identify a tagged object. It is used in several commercial and industrial applications, from tracking items along a supply chain to keeping track of items checked out of a library.

A wireless LAN is a wireless computer network that links two or more devices using wireless communication to form a local area network within a limited area such as a home, school, computer laboratory, campus, or office building.

Source - https://microbewiki.kenyon.edu/index.php/Biofilms_and_Human_Implants


17. consider the following statements:

  1. Biofilms can form on medical implants within human tissues
  2. Biofilms can form on food and food processing surfaces.
  3. Biofilms can exhibit antibiotic resistance.

Which of the statements given above are correct?

  1. 1 and 2 only
  2. 2 and 3 only
  3. 1 and 3 only
  4. 1, 2 and 3

Sol. Ans.(d). A biofilm is an assemblage of surface-associated microbial cells that is enclosed in an extracellular polymeric substance matrix. Van Leeuwenhoek, using his simple microscopes, first observed microorganisms on tooth surfaces and can be credited with the discovery of microbial biofilms. These often form on the surfaces of implanted medical devices such as urinary and venous catheters, breast implants, and pacemakers. So, 1 is correct. Biofilms are formed on all types of surfaces existing in food plants ranging from plastic, glass, metal, cement, to wood and food products. So, 2 is also correct. Biofilm structures show maximum resistance to antibiotics in the mature stage. So, 3 is also correct.

Source 1 - https://www.intechopen.com/chapters/70036

Source 2 - https://www.tandfonline.com/doi/full/10.4161/viru.23724


18. Consider the following statements in respect of probiotics:

  1. Probiotics are made of both bacteria and yeast.
  2. The organisms in probiotics are found in foods we ingest but they do not naturally occur in our gut.
  3. Probiotics help in the digestion of milk sugars.

Which of the statements given above is/are correct?

  1. 1 only
  2. 2 only
  3. 1 and 3
  4. 2 and 3

Sol. Ans.(c). Probiotics are a mixture of live bacteria and/or yeast that lives in your body. Probiotics are good bacteria that help keep you healthy. So, 1 is correct, options (b) and (c) are ruled out. Many probiotic bacterial strains also produce beta-galactosidase, an enzyme that acts like lactase and can help to break down lactose in the gut. So, 3 is also correct.

Source 1 - https://my.clevelandclinic.org/health/articles/14598-probiotics

Source 2 -
https://www.optibacprobiotics.com/professionals/latest-research/general-health/probiotic-supplements-help-lactose-intolerance#:~:text=But%20it's%20also%20the%20action,like%20lactase%20and%20can%20help


19. In the context of vaccines manufactured to prevent COVID-19 pandemic, consider the following statements:

  1. The Serum Institute of India produced COVID-19 vaccine named Covishield using mRNA platform.
  2. Sputnik V vaccine is manufactured using vector based platform.
  3. COVAXIN is an inactivated pathogen based vaccine.

Which of the statements given above are correct?

  1. 1 and 2 only
  2. 2 and 3 only
  3. 1 and 3 only
  4. 1, 2 and 3

Sol. Ans.(b). The Oxford–AstraZeneca COVID-19 vaccine, sold under the brand names Covishield and Vaxzevria among others, is a viral vector vaccine for prevention of COVID-19. So, statement 1 is not correct, only possible option is (b). Three options were ruled out. We can consider this question a Sitter!

( This question was directly covered in PT’s Powerpack Course – enquire and enroll here - https://bit.ly/upscias )


20. If a major solar storm (solar flare)-reaches the Earth, which of the following are the possible effects on the Earth?

  1. GPS and navigation systems could fail.
  2. Tsunamis could occur at equatorial regions.
  3. Power grids could be damaged,
  4. Intense auroras could occur over much of the Earth.
  5. Forest fires could take place over much of tile planet.
  6. Orbits of the satellites could be disturbed.
  7. Shortwave radio communication of the aircraft flying over Polar Regions could be interrupted.

Select the correct answer using the code given below;

  1. 1, 2, 4 and 5 only
  2. 2, 3, 5, 6 and 7 only
  3. 1, 3, 4, 6 and 7 only
  4. 1, 2, 3, 4, 5, 6 and 7

Sol. Ans.(c). Tsunamis occur due to the plate tectonics movements. The incidence of solar flares hitting Earth will not result in tsunamis as they are both just not interlinked. So, statement 2 should not be the part of correct option. Only possible option is (c). This was an easy one if you knew this fact.

( This question was directly covered in PT’s Powerpack Course – enquire and enroll here - https://bit.ly/upscias )

Source - https://www.space.com/solar-flares-effects-classification-formation


21. Consider the following statements:

  1. Pursuant to the report of H.N. Sanyal Committee, the Contempt of Courts Act, 1971 was passed.
  2. The Constitution of India empowers the Supreme Court and the High Courts to punish for contempt of themselves.
  3. The Constitution of India defines Civil Contempt and Criminal Contempt.
  4. In India, the Parliament is vested with the powers to make laws on Contempt of Court.

Which of the statements given above is/are correct?

  1. 1 and 2 only
  2. 1, 2 and 4
  3. 3 and 4 only
  4. 3 only

Sol. Ans.(a). In 1961, a committee headed by H. N. Sanyal, an Additional Solicitor General for the Government of India, was appointed to examine the application of contempt laws in India. On recommendations of this committee the Contempt of Courts Act, 1971 was passed. So 1 is correct.

Focus on statement 3. It is wrong, as these two types were defined by the 1971 Act and not the Constitution. So (a) or (b) may be our answer.

Final answer is (a).

Source 1 - https://www.legalserviceindia.com/article/l255-Contempt-of-Court.html

Source 2 - https://www.jagranjosh.com/general-knowledge/explained-power-to-punish-for-contempt-of-court-cannot-be-abridged-by-parliament-sc-1633681057-1

Source 3 - https://indianculture.gov.in/report-committee-contempt-courts-1963


22. With reference to India, consider the following statements:

  1. Government law officers and legal firms are recognised as advocates, but corporate lawyers and patent attorneys are excluded from recognition as advocates.
  2. Bar Councils have the power to lay down the rules relating to legal education and recognition of law colleges.

Which of the statements given above is/are correct?

  1. 1 only
  2. 2 only
  3. Both 1 and 2
  4. Neither 1 nor 2

Sol. Ans.(b). A patent attorney is an advocate who practices IP law and may or may not be a patent agent. Further, a patent attorney must be registered with the Bar Council of India as an advocate. An advocate is not eligible to write the Indian patent agent exam if he is not a graduate of a scientific domain. (It is true that corporate lawyers have to surrender their “sanad” and cannot practice as advocates)

The Bar Council of India visits and inspects Universities/Law colleges in the country as part of its statutory function of promoting legal education and laying down standards in consultation with the Universities in India and the State Bar Councils.

So 1 is not correct and 2 is correct.

Source 1 -
http://www.barcouncilofindia.org/about/legal-education/#:~:text=The%20Bar%20Council%20of%20India%20visits%20and%20inspects%20Universities%2FLaw,and%20the%20State%20Bar%20Councils.

Source 2 -
https://www.origiin.com/2021/01/22/career-in-ipr-for-advocates/#:~:text=A%20patent%20attorney%20on%20the,graduate%20of%20a%20scientific%20domain.


23. Consider the following statements:

  1. A bill amending the Constitution requires a prior recommendation of the President of India.
  2. When a Constitution Amendment Bill is presented to the President of India, it is obligatory for the President of India to give his/her assent.
  3. A Constitution Amendment .Bill must be passed by both the Lok Sabha and the Rajya Sabha by a special majority and there is no provision for joint sitting.

Which of the statements given above are correct?

  1. 1 and 2 only
  2. 2 only
  3. 1 and 3 only
  4. 1, 2 and 3

Sol. Ans.(b). Constitutional Amendment process is as laid down in Article 368 –

  1. An amendment of the Constitution can be initiated only by the introduction of a bill for the purpose in either House of Parliament (Lok Sabha & Rajya Sabha) and not in the state legislatures.
  2. The bill can be introduced either by a minister or by a private member and does not require prior permission of the president.
  3. The bill must be passed in each House by a special majority, that is, a majority (that is, more than 50 per cent) of the total membership of the House and a majority of two-thirds of the members of the House present and voting.
  4. Each House must pass the bill separately.
  5. In case of a disagreement between the two Houses, there is no provision for holding a joint sitting of the two Houses for the purpose of deliberation and passage of the bill.
  6. If the bill seeks to amend the federal provisions of the Constitution, it must also be ratified by the legislatures of half of the states by a simple majority, that is, a majority of the members of the House present and voting.
  7. After duly passed by both the Houses of Parliament and ratified by the state legislatures, where necessary, the bill is presented to the president for assent.
  8. The president must give his assent to the bill. He can neither withhold his assent to the bill nor return the bill for reconsideration of the Parliament.
  9. After the president’s assent, the bill becomes an Act (i.e., a constitutional amendment act) and the Constitution stands amended in accordance with the terms of the Act.

Now note that since statement 1 is wrong, three options are ruled out. Hence this is a sitter! ( This question was covered in PT’s Powerpack Course – enquire and enroll here - https://bit.ly/upscias )


24. Consider the following statements:

  1. The Constitution of India classifies the ministers into four ranks viz. Cabinet Minister, Minister of State with Independent Charge, Minister of State and Deputy Minister.
  2. The total number of ministers in the Union Government, including the Prime Minister, shall not exceed 15 percent of the total number of members in the Lok Sabha.
  3. A Constitution Amendment .Bill must be passed by both the Lok Sabha and the Rajya Sabha by a special majority and there is no provision for joint sitting.

Which of the statements given above is/are correct?

  1. 1 only
  2. 2 only
  3. Both 1 and 2
  4. Neither 1 nor 2

Sol. Ans.(b). As per the 91st Amendment Act, the total number of Ministers, including the Prime Minister, in the Council of Ministers shall not exceed fifteen per cent. Of the total number of members of the House of the People. Hence, (a) and (d) are ruled out. The constitution of India does not classify any ranks of ministers like Cabinet Minister, Minister of State with independent in charge, Minister of State or Deputy Minister. So, only statement 2 is correct. (In fact, there are only three types of Ministers and no post called “Deputy Minister”). So 1 is incorrect.

( This question was covered in PT’s Powerpack Course – enquire and enroll here - https://bit.ly/upscias )

Source: https://legislative.gov.in/sites/default/files/amend91.pdf


25. "Which of the following is/are the exclusive power (s) of Lok Sabha?

  1. To ratify the declaration of Emergency
  2. To pass a motion of no-confidence against the Council of Ministers
  3. To impeach the President of India

Select the correct answer using the code given below:

  1. 1 and 2
  2. 2 only
  3. 1 and 3
  4. 3 only

Sol. Ans.(b). The Lok Sabha has only some powers that make it more powerful than the Rajya Sabha.

  1. Motions of no confidence against the government can be introduced and passed in the Lok Sabha, but not in the Rajya Sabha. This is the most crucial difference, as the Council of Ministers is responsible to the Lok Sabha.
  2. Money bills can only be introduced in the Lok Sabha
  3. The budget is presented in the Lok Sabha by the Finance Minister in the name of the President of India.
  4. In matters about non-financial (ordinary) bills, after the bill has been passed by the House where it was originally tabled (Lok Sabha or Rajya Sabha), it is sent to the other house, where it may be kept for a maximum period of 6 months.
  5. Equal Powers with the Rajya Sabha in initiating and passing any Bill for Constitutional Amendment (by a majority of the total membership of the House and at least two-thirds majority of the members present and voting).
  6. Equal Powers with the Rajya Sabha in initiating and passing a motion for the impeachment of the President (by two-thirds of the membership of the House).
  7. Equal Powers with the Rajya Sabha in the impeachment process (initiating and passing a motion for the removal) of the judges of the Supreme Court and the state High Courts (by a majority of the membership of the House and at least two-thirds majority of the members present and voting), who then can be removed by the President of India.
  8. Equal Powers with the Rajya Sabha in initiating and passing a resolution declaring war or national emergency (by two-thirds majority) or constitutional emergency (by simple majority) in a state.
  9. If the Lok Sabha is dissolved before or after the declaration of a National Emergency, the Rajya Sabha becomes the sole Parliament. It cannot be dissolved. This is a limitation on the Lok Sabha.

( This question was covered in PT’s Powerpack Course – enquire and enroll here - https://bit.ly/upscias )


26. With reference to anti-defection law in India, consider the following statements:

  1. The law specifies that a nominated legislator cannot join any political party within six months of being appointed to the House.
  2. The law does not provide any time-frame within which the presiding officer has to decide a defection case.

Which of the statements given above is/are correct?

  1. 1 only
  2. 2 only
  3. Both 1 and 2
  4. Neither 1 nor 2

Sol. Ans.(b). The Anti-defection law received the President's approval on 15 February 1985 and the Act came into effect on 18 March 1985. The law laid out the process for disqualifying an elected member for the remaining term, who defected either by resigning or by defying the party leadership and being absent on a crucial vote. In other word, the anti-defection law punishes individual Members of Parliament (MPs)/MLAs for leaving one party for another.

Grounds of disqualification

  1. If an elected member voluntarily gives up his membership of a political party.
  2. If he votes or abstains from voting in such House contrary to any direction issued by his political party or anyone authorised to do so, without obtaining prior permission.
  3. As a pre-condition for his disqualification, his abstention from voting should not be condoned by his party or the authorised person within 15 days of such incident.
  4. If any independently elected member joins any political party.
  5. If any nominated member joins any political party after the expiry of six months

However, the law does not provide a time-frame within which the presiding officer has to decide a defection case.


27. Consider the following statements:

  1. Attorney General of India and Solicitor General of India are the only officers of the Government who are allowed to participate in the meetings of the Parliament of India.
  2. According to the Constitution of India, the Attorney General of India submits his resignation when the Government which appointed him resigns,

Which of the statements given above is/are correct?

  1. 1 only
  2. 2 only
  3. Both 1 and 2
  4. Neither 1 nor 2

Sol. Ans.(d). Article 76 of the constitution mentions that he/she is the highest law officer of India. As a chief legal advisor to the government of India, he advises the union government on all legal matters. He also is the primary lawyer representing Union Government in the Supreme Court of India. The Attorney General, like an Advocate General of a State, is not supposed to be a political appointee, in spirit, but this is not the case in practice.

President of India appoints a person who is qualified for the post of Supreme Court Judge. Attorney General is appointed by the President on the advice of the government. There are the following qualifications - He should be an Indian Citizen; He must have either completed 5 years in High Court of any Indian state as a judge or 10 years in High Court as an advocate; He may be an eminent jurist too, in the eye of the President.

There is no fixed term for the Attorney General of India. The Constitution mentions no specified tenure of Attorney General. Similarly, the Constitution also does not mention the procedure and ground of his removal.

He can be removed by the President at any time. He can quit by submitting his resignation only to the President. Since he is appointed by the President on the advice of the Council of Ministers, conventionally he is removed (by President) when the council is dissolved or replaced.

Attorney General of India can attend the meetings of both houses of parliament while not being a member of either.

The Solicitor General of India is subordinate to the Attorney General for India. He is the second law officer of the country, and assists the Attorney General, and is assisted by Additional Solicitors General for India. They can not attend the meetings of parliaments. So, both statements are not true.


28. With reference to the writs issued by the Courts in India, consider the following statements:

  1. Mandamus will not lie against a private organisation unless it is entrusted with a public duty.
  2. Mandamus will not lie against a Company even though it may be a Government Company.
  3. Any public minded person can be a petitioner to move the Court to obtain the writ of Quo Warranto.

Which of the statements given above are correct?

  1. 1 and 2 only
  2. 2 and 3 only
  3. 1 and 3 only
  4. 1, 2 and 3

Sol. Ans.(c). ‘Mandamus’ means ‘we command’. It is issued by an empowered Court to direct a public authority to perform the legal duties which it has not or refused to perform. It can be issued by the Court against a public official, public corporation, tribunal, inferior court or the government. It cannot be issued against a private individual or body, the President or Governors of States or against a working Chief Justices.

It cannot be issued in the following circumstances - The duty in question is discretionary and not mandatory. For the performance of a non-statutory function. Performance of the duty involves rights of purely private nature. Where such direction involves violation of any law.

Under Article 226 of the Constitution, a Writ of Mandamus can be issued even against a private authority in respect of the public functions being discharged by the said authority. A writ court can by its decision direct a private authority to correct or enforce discharge of public functions. So, 1 is correct.

About issuing mandamus to government companies, the law seems to be in a state of confusion! A few High Courts have thought that the writ could apply even to a government company, but few others have held that the writ may not be issued to a statutory corporation like Life Insurance Corporation. The position in relation to the above bodies is examined seriatim (a Latin word meaning consecutively).

The writ of Quo Warranto is issued by the courts against a private person when he assumes an office on which he has no right. Quo Warranto literally means ‘by what authority’ and it is an effective measure to prevent people from taking over public offices. A citizen can claim a writ of quo warranto and he stands in the position of a relater. He need not have any special interest or personal interest.

Source 1 : https://www.jstor.org/stable/43950186

Source 2 : https://indiankanoon.org/doc/1760972/#:~:text=Under%20the%20said%20Article%2C%20a,enforce%20discharge%20of%20public%20functions

Source 3: https://main.sci.gov.in/pdf/SupremeCourtReport/2012_v10_piv.pdf


29. With reference to Ayushman Bharat Digital Mission, consider the following statements:

  1. Private and public hospitals must adopt it.
  2. As it aims to achieve universal health coverage, every citizen of India should be part of it ultimately.
  3. It has seamless portability across the country.

Which of the statements given above is/are correct?

  1. 1 and 2 only
  2. 3 only
  3. 1 and 3 only
  4. 1, 2 and 3

Sol. Ans.(b). Ayushman Bharat Digital Mission aims to provide digital health IDs for all Indian citizens to help hospitals, insurance firms, and citizens access health records electronically when required. A health ID will be issued for every citizen that will also work as their health account. This health account will contain details of every test, every disease, the doctors visited, the medicines taken and the diagnosis. Health ID is free of cost, voluntary. It will help in doing analysis of health data and lead to better planning, budgeting and implementation for health programs.

The other major component of the programme is creating a Healthcare Professionals’ Registry (HPR) and Healthcare Facilities Registry (HFR), allowing easy electronic access to medical professionals and health infrastructure. The HPR will be a comprehensive repository of all healthcare professionals involved in delivering healthcare services across both modern and traditional systems of medicine. The HFR database will have records of all the country’s health facilities. (public and private)

Participation in ABDM is voluntary including for citizens. Participation of a healthcare facility or an institution is also voluntary and shall be taken by the respective management (government or private management). However, once the management decides to register the respective healthcare facility/institution in ABDM, it is essential for all the healthcare professionals serving the said facility/institution to register in Healthcare Professionals Registry so that the institution can become fully integrated with the National Digital Health Ecosystem (NDHE).

So, 1 is not correct, only possible option is (b).

Source : https://abdm.gov.in/faq


30. With reference to Deputy Speaker of Lok Sabha, consider the following statements:

  1. As per the Rules of Procedure and Conduct of Business in Lok Sabha, the election of Deputy Speaker shall be held on such date as the Speaker may fix.
  2. There is a mandatory provision that the election of a candidate as Deputy Speaker of Lok Sabha shall be from either the principal opposition party or the ruling party.
  3. The Deputy Speaker has the same power as of the Speaker when presiding over the sitting of the House and no appeal lies against his rulings.
  4. The well established parliamentary practice regarding the appointment of Deputy Speaker is that the motion is moved by the Speaker and duly seconded by the Prime Minister.

Which of the statements given above are correct?

  1. 1 and 3 only
  2. 1, 2 and 3
  3. 3 and 4 only
  4. 2 and 4 only

Sol. Ans.(a). The Deputy Speaker of the Lok Sabha is the second highest ranking legislative officer of the Lok Sabha, the lower house of the Parliament of India.

The Deputy Speaker is also elected by the Lok Sabha from amongst its members right after the election of the Speaker has taken place. The date of election of the Deputy Speaker is fixed by the Speaker (date of election of the Speaker is fixed by the President). So, 1 is correct.

The Deputy Speaker performs the duties of the Speaker’s office when it is vacant. He/She also acts as the Speaker when the latter is absent from the sitting of the House. He/She also presides over the joint sitting of both the Houses of Parliament, in case the Speaker is absent from such a sitting. The Deputy Speaker has one special privilege, that is, whenever he/she is appointed as a member of a parliamentary committee, he/she automatically becomes its chairman. So, 3 is correct.

1 and 3 are correct – so only possible option is (a).


31. "Rapid Financing Instrument" and "Rapid Credit Facility" are related to the provisions of lending by which one of the following?

  1. Asian Development Bank
  2. International Monetary Fund
  3. United Nations Environment Programme Finance Initiative
  4. World Bank

Sol. Ans.(b). First, the Rapid Financing Instrument (RFI) - It provides rapid financial assistance, which is available to all member countries facing an urgent balance of payments need. The RFI was created as part of a broader reform to make the IMF’s financial support more flexible to address the diverse needs of member countries. The RFI replaced the IMF’s previous emergency assistance policy and can be used in a wide range of circumstances.

Rapid Credit Facility (RCF) – It provides rapid concessional financial assistance to low-income countries (LICs) facing an urgent balance of payments (BoP) need with no ex post conditionality where a full-fledged economic program is neither necessary nor feasible. The RCF was created under the Poverty Reduction and Growth Trust (PRGT) as part of a broader reform to make the Fund’s financial support more flexible and better tailored to the diverse needs of LICs, including in times of crisis.

Source : https://www.imf.org/en/About/Factsheets/Sheets/2016/08/02/19/55/Rapid-Financing-Instrument

( This question was covered in PT’s Powerpack Course – enquire and enroll here - https://bit.ly/upscias )


32. With reference to the Indian economy, consider the following statements:

  1. An increase in Nominal Effective Exchange Rate (NEER) indicates the appreciation of rupee.
  2. An increase in the Real Effective Exchange Rate (REER) indicates an improvement in trade competitiveness.
  3. An increasing trend in domestic inflation relative to inflation in other countries is likely to cause an increasing divergence between NEER and REER.

Which of the above statements are correct?

  1. 1 and 2 only
  2. 2 and 3 only
  3. 1 and 3 only
  4. 1, 2 and 3

Sol. Ans.(c). The NEER (nominal effective exchange rate) is a measure of value of a currency against a weighted average of several foreign currency. An increase in NEER indicates appreciation of rupee. So, statement 1 is correct.

An increase in REER implies that exports become more expensive and imports become cheaper; therefore, an increase indicates a loss in trade competitiveness. So, statement 2 is incorrect.

The NEER is the weighted geometric average of the bilateral nominal exchange rates of the home currency in terms of foreign currencies. The REER is the weighted average of NEER adjusted by the ratio of domestic price to foreign prices. Increasing trend in domestic inflation relative to inflation in other countries creates a divergence in NEER and REER. So, 3 is correct.

This was a very tough question, and could have been avoided unless studied thoroughly earlier.

( This question was covered in PT’s Powerpack Course – enquire and enroll here - https://bit.ly/upscias )


33. With reference to the Indian economy, consider the following statements:

  1. If the inflation is too high. Reserve Bank of India (RBI) is likely to buy government securities.
  2. If the rupee is rapidly depreciating, RBI is likely to sell dollars in the market.
  3. If interest rates in the USA or European Union were to fall, that is likely to induce RBI to buy dollars.

Which of the statements given above are correct?

  1. 1 and 2 only
  2. 2 and 3 only
  3. 1 and 3 only
  4. 1, 2 and 3

Sol. Ans.(b). When the inflation is high RBI tries to reduce the liquidity from the market by selling Government securities to the public via open market operations (OMO). So, statement 1 is incorrect. So options (a), (c) and (d) are gone. Answer has to be (b), so this is a Sitter!

Now – “Rupee depreciation” means a decrease in value of Rupee with respect to Dollar. In a free floating exchange rate regime, depreciation takes place when the demand for Dollar is more than the supply. RBI is likely to sell dollars in the market, if rupee rapidly depreciates, to increase the supply of the dollar (and stop the rapid depreciation). So, statement 2 is correct.

When the interest rates in US and EU fall, there will be an inflow of dollars from the Indian market (an effect of “cheap money abroad”), leading to appreciation of the rupee. That will hurt Indian exporters and IT firms (and also NRIs abroad!). To reduce the supply of dollars in the economy, RBI will likely buy dollars from the market. So, statement 3 is also correct.

( This question was covered in PT’s Powerpack Course – enquire and enroll here - https://bit.ly/upscias )


34. With reference to the "G20 Common Framework", consider the following statements:

  1. It is an initiative endorsed by the “G20 together with the Paris Club.
  2. It is an initiative to support Low Income Countries with unsustainable debt.

Which of the statements given above is/are correct?

  1. 1 only
  2. 2 only
  3. Both 1 and 2
  4. Neither 1 nor 2

Sol. Ans.(c). Initiative endorsed by G20 together with Paris Club and is an initiative to support low income countries with unsustainable debt. Both statements are correct. Please refer to the source more information.

Source : https://blogs.imf.org/2021/12/02/the-g20-common-framework-for-debt-treatments-must-be-stepped-up/


35. With reference to the Indian economy, what are the advantages of "Inflation-Indexed Bonds (IIBs)"?

  1. Government can reduce the coupon rates on its borrowing by way of IIBs.
  2. IIBs provide protection to the investors from uncertainty regarding inflation.
  3. The interest received as well as capital gains on IIBs are not taxable.

Which of the statements given above are correct?

  1. 1 and 2 only
  2. 2 and 3 only
  3. 1 and 3 only
  4. 1, 2 and 3

Sol. Ans.(a). The government can reduce the coupon rates on its borrowings by way of IIBs. Inflation indexed bonds provides protection to investors from uncertainty regarding inflation. Extant tax provisions will be applicable on interest payment and capital gains on IIBs. So 1 and 2 are correct and 3 is not. (This question requires a student to know about multiple types of financial instruments thoroughly).

Source : https://m.rbi.org.in/Scripts/FAQView.aspx?Id=91


36. With reference to foreign-owned e-commerce firms operating in India, which of the following statements is/are correct?

  1. They can sell their own goods in addition to offering their platforms as market-places.
  2. The degree to which they can own big sellers on their platforms is limited.

Select the correct answer using the code given below:

  1. 1 only
  2. 2 only
  3. Both 1 and 2
  4. Neither 1 nor 2

Sol. Ans.(b). The issue of “whether FDI-funded e-commerce firms can hold inventory on their own marketplace or not” has seen many policy flip-flops over the years.

India recently tightened the foreign direct investment (FDI) rules for ecommerce to check companies set up by the online marketplaces from trading on their own platform. The rules barred online marketplaces from holding inventory of their own or influencing the price of goods on their platforms. They also prohibited group companies or entities in which marketplaces have control of inventory to sell on their platforms, among other things. (many of these rules have existed for many years)

Under the FDI Policy in India, the Equity/FDI cap on e-commerce activities is set at 100% through the automatic route. However, e-commerce startups and entities should engage only in the Business to Business (B2B) e-commerce and not in the Business to Consumer (B2C) e-commerce. (doing that would actually make no sense for firms like Flipkart or Amazon!)

The FDI Policy allows 100% FDI under the automatic route for the marketplace model of e-commerce activities. However, FDI is not permitted for the inventory-based model of e-commerce activities.

The marketplace-based model of e-commerce means providing an information technology platform by an e-commerce startup or entity on a digital and electronic network, acting as a facilitator between the buyer and the seller.

The inventory-based model of e-commerce means e-commerce activities where the inventory of goods and services is owned by an e-commerce startup or entity and is sold to the consumers directly.

So, statement 1 is not correct, but 2 is correct.

Source :
https://economictimes.indiatimes.com/tech/tech-bytes/2021-year-in-review-the-chaos-and-confusion-around-indias-ecommerce-rules/articleshow/88614007.cms?from=mdr

Source : https://cleartax.in/s/fdi-regulations-e-commerce-startups


37. Which of the following activities constitute real sector in the economy?

  1. Farmers harvesting their crops
  2. Textile mills converting raw cotton into fabrics
  3. A commercial bank lending money to a trading company
  4. A corporate body issuing Rupee Denominated Bonds overseas

Select the correct answer using the code given below:

  1. 1 and 2 only
  2. 2, 3 and 4 only
  3. 1, 3 and 4 only
  4. 1, 2, 3 and 4

Sol. Ans.(a). The real sector of the economy consists of enterprises (non-financial corporations), households and nonprofit institutions serving households. The statements 3 and 4 mention the financial corporations, and so they cannot be considered as real sector in the economy.

( This question was covered in PT’s Powerpack Course – enquire and enroll here - https://bit.ly/upscias )

Source : https://www.cba.am/Storage/EN/publications/statistics/monetary_stat_manual/sectors.pdf


38. Which one of the following situations best reflects "Indirect Transfers" often talked about in media recently with reference to India?

  1. An Indian company investing in a foreign enterprise and paying taxes to the foreign country on the profits arising out of its investment
  2. A foreign company investing in India and paying taxes to the country of its base on the profits arising out of its investment
  3. An Indian company purchases tangible assets in a foreign country and sells such assets after their value increases and transfers the proceeds to India
  4. A foreign company transfers shares and such shares derive their substantial value from assets located in India

Sol. Ans.(d). Indirect transfers refer to situations where when foreign entities own shares or assets in India, and the shares of such foreign entities are transferred instead of a direct transfer of the underlying assets in India. This question was relevant in context of the Vodafone share transfer and retrospective taxation controversy.

Source :
https://kluwerlawonline.com/journalarticle/Intertax/45.10/TAXI2017057#:~:text=Indirect%20transfers%20refer%20to%20situations,the%20underlying%20assets%20in%20India


39. With reference to the expenditure made by an organisation or a company, which of the following statements is/are correct?

  1. Acquiring new technology is capital expenditure.
  2. Debt financing is considered capital expenditure, while equity financing is considered revenue expenditure.

Select the correct answer using the code given below:

  1. 1 only
  2. 2 only
  3. Both 1 and 2
  4. Neither 1 nor 2

Sol. Ans.(a). Capital expenditure is the money spent (by the government or firms) on the development of machinery, equipment, building, technology, health facilities, education, etc. It also includes the expenditure incurred on acquiring fixed assets like land and investments that give profits or dividend in future. You can do capital expenditure by debt financing, or through other methods.

Revenue expenditure is one that neither creates assets nor reduces any liability of the government. Salaries of employees, interest payment on past debt, subsidies, pension, etc. fall under the category of revenue expenditure. It is recurring in nature. So equity financing cannot be Revenue Expenditure.

So, only 1 is correct.

Source 1 : https://www.investopedia.com/terms/c/capitalexpenditure.asp#:~:text=Capital%20expenditures%20(CapEx)%20are%20funds,or%20investments%20by%20a%20company

Source 2 : https://www.business-standard.com/about/what-is-capital-expenditure


40. With reference to the Indian economy, consider the following statements:

  1. A share of the household financial savings goes towards government borrowings.
  2. Dated securities issued at market-related rates in. auctions form a large component of internal debt.

Which of the above statements is/are correct?

  1. 1 only
  2. 2 only
  3. Both 1 and 2
  4. Neither 1 nor 2

Sol. Ans.(c). How critical are household savings for government borrowing? - The importance of household savings can be gauged from the fact that a household is the only segment that is the net financial surplus sector in the economy; all other segments, namely public sector, private sector and general government, are in deficit. Also, the share of marketable securities in internal debt stood at 84.4 per cent at end-March 2019, slightly lower than 86.1 per cent at end-March 2018. (Dated securities means long-term G-secs)

( This question was covered in PT’s Powerpack Course – enquire and enroll here - https://bit.ly/upscias )

Source 1 : https://www.financialexpress.com/opinion/how-critical-are-household-savings-for-government-borrowing/1542170/

Source 2 : https://dea.gov.in/sites/default/files/Status%20Paper%20on%20Government%20Debt.pdf


41. Among the following crops, which one is the most important anthropogenic source of both methane and nitrous oxide?

  1. Cotton
  2. Rice
  3. Sugarcane
  4. Wheat

Sol. Ans.(b). Important anthropogenic sources of biogenic methane are wet rice fields, cattle, animal waste, landfills and biomass burning. Important anthropogenic sources of biogenic nitrous oxide are land-use change, fertilizer production and use and manure application. An excellent question combining environment & ecology, and agriculture.

Source: https://link.springer.com/article/10.1007/BF00748940


42. "System of Rice Intensification" of cultivation, in which alternate wetting and drying of rice fields is practiced, results in:

  1. Reduced seed requirement
  2. Reduced methane production
  3. Reduced electricity consumption

Select the correct answer using the code given below:

  1. 1 and 2 only
  2. 2 and 3 only
  3. 1 and 3 only
  4. 1, 2 and 3

Sol. Ans.(d). The system of Rice Intensification involves cultivating rice with as much organic manure as possible. In SRI paddy cultivation, less quantity of seeds - 2 kg / acre is required. Research has shown that intermittent paddy irrigation by SRI or AWD reduced methane emissions by between 22% and 64%.

Under SRI, the process starts with young seedlings planted singly at wider spacing in a square pattern; and with intermittent irrigation that keeps the soil moist but not inundated, and frequent inter cultivation with weeder that actively. As less irrigation is required, it results in reduced electricity consumption. So, all the three statements are correct.

Source 1 : https://vikaspedia.in/agriculture/best-practices/sustainable-agriculture/crop-management/sri-2013-new-method-of-growing-rice

Source 2 : http://sri.ciifad.cornell.edu/index_files/ClimateChangeMitigation.pdf


43. Which one of the following lakes of West Africa has become dry and turned into a desert?

  1. Lake Victoria
  2. Lake Faguibine
  3. Lake Oguta
  4. Lake Volta

Sol. Ans.(b). Lake Victoria is in East Africa (Tanzania, Uganda and Bordering on Kenya). Lake Faguibine is a lake in Mali on the southern edge of the Sahara Desert situated 80 km west of Timbuktu and 75 km north of the Niger River to which it is connected by a system of smaller lakes and channels. In years when the height of the annual flood of the river is sufficient, water flows from the river into the lake. Since the Sahel drought of the 1970s and 1980s the lake has been mostly dry. Water has only rarely reached the lake and even when it has done so, the lake has been only partially filled with water. This has caused a partial collapse of the local ecosystem.


44. Gandikota canyon of South India was created by which one of the following rivers?

  1. Cauvery
  2. Manjira
  3. Pennar
  4. Tungabhadra

Sol. Ans.(c). Gandikota is a small village in the Kadapa district of Andhra Pradesh that is known for its spectacular gorge formed by the river Pennar that cuts through the Erramala hills. This picturesque piece of nature’s architecture has come to be known as the Hidden Grand Canyon of India by travelers who visit it and get bewitched by its beauty. A very specific question on geography.


45. Consider the following pairs:

Which of the pairs given above Ware correctly matched?

  1. 1 and 2
  2. 2 only
  3. 1 and 3
  4. 3 only

Sol. Ans.(b). Namcha Barwa is located in the Eastern Himalayas - Arunachal Pradesh. Garhwal Himalayas is in Uttarakhand. Nanda Devi is located in the state of Uttarakhand specifically in the Kumaon Himalayas. Nokrek is located in the state of Meghalaya in the Garo Hills. So only 2 is correct, not 1 and 3.

( This question was covered in PT’s Powerpack Course – enquire and enroll here - https://bit.ly/upscias )


46. The term "Levant" often heard in the news roughly corresponds to which of the following regions?

  1. Region along the eastern Mediterranean shores
  2. Region along North African shores stretching from Egypt to Morocco
  3. Region along Persian Gulf and Horn of Africa
  4. The entire coastal areas of Mediterranean Sea

Sol. Ans.(a). The Levant is an approximate historical geographical term referring to a large area in the Eastern Mediterranean region of Western Asia.

( This question was covered in PT’s Powerpack Course – enquire and enroll here - https://bit.ly/upscias )


47. Consider the following countries:

  1. Azerbaijan
  2. Kyrgyzstan
  3. Tajikistan
  4. Turkmenistan
  5. Uzbekistan

Which of the above have borders with Afghanistan?

  1. 1, 2 and 5 only
  2. 1, 2, 3 and 4 only
  3. 3, 4 and 6 only
  4. 1, 2, 3, 4 and 5

Sol. Ans.(c). Out of the given options, the bordering countries of Afghanistan are Turkmenistan, Uzbekistan and Tajikistan. Note that Kyrgyzstan is not bordering it. That eliminates three options straight (a, b and d). A sitter actually!

( This question was covered in PT’s Powerpack Course – enquire and enroll here - https://bit.ly/upscias )


48. With reference to India, consider the following statements:

  1. Monazite is a source of rare earths.
  2. Monazite contains thorium.
  3. Monazite occurs naturally in the entire Indian coastal sands in India.
  4. In India, Government bodies only can process or export monazite.

Which of the statements given above are correct?

  1. 1, 2 and 3 only
  2. 1, 2 and 4 only
  3. 3 and 4 only
  4. 1, 2, 3 and 4

Sol. Ans.(b). Monazite is a primarily reddish-brown phosphate mineral that contains rare-earth elements. Thorium content of monazite is variable and sometimes can be up to 20–30%. Monazite from certain carbonatites or from Bolivian tin ore veins is essentially thorium-free. However, commercial monazite sands typically contain between 6 and 12% thorium oxide. Monazite occurs naturally in the entire eastern coast and Kerala coastal sands in India (that can’t be considered as entire western coastline). So 1 and 2 are true but 3 is not. If 3 is wrong, then three options are ruled out instantly (a, c and d). Also, only state-run Indian Rare Earths Limited (IREL) can produce and process monazite, a beach sand mineral containing thorium. The question is almost a sitter.

( This question was covered in PT’s Powerpack Course – enquire and enroll here - https://bit.ly/upscias )

Source 1 :
https://economictimes.indiatimes.com/industry/indl-goods/svs/metals-mining/no-private-licence-given-for-monazite-production-dae/articleshow/16900031.cms?utm_source=contentofinterest&utm_medium=text&utm_campaign=cppst

Source 2 : https://en.wikipedia.org/wiki/Monazite


49. In the northern hemisphere, the longest day of the year normally occurs in the:

  1. First half of the month of June
  2. Second half of the month of June
  3. First half of the month of July
  4. Second half of the month of July

Sol. Ans.(c). The longest day for those living north of the Equator is June 21. In technical terms, this day is referred to as the Summer Solstice, the longest day of the summer season. It occurs when the sun is directly over the Tropic of Cancer. A standard question from geography.

( This question was covered in PT’s Powerpack Course – enquire and enroll here - https://bit.ly/upscias )


50. Consider the following pairs:

How many pairs given above are correctly matched?

  1. Only one pair
  2. Only two pairs
  3. Only three pairs
  4. All four pairs

Sol. Ans.(b). Hokera Wetland – Jammu and Kashmir, Renuka Wetland – Himachal Pradesh, Rudrasagar lake – Tripura, Sasthamkotta lake – Kerala. Only two pairs are correct. Now this question required you to actually remember all wetlands and their locations in India!

( This question was covered in PT’s Powerpack Course – enquire and enroll here - https://bit.ly/upscias )


51. Consider the following pairs:

How many pairs given above are correctly matched?

  1. Only one pair
  2. Only two pairs
  3. Only three pairs
  4. All four pairs

Sol. Ans.(b). This is a question from “Ancient and Medieval history”. Dhauli is located in Odisha. Erragudi or Yerragudi is located in Andhra Pradesh. Jaugada is located in Odisha. Kalsi is located in Uttarakhand. Only two pairs are correct. A tough question as it requires you to remember all rock edits of Emperor Ashoka.

( This question was covered in PT’s Powerpack Course – enquire and enroll here - https://bit.ly/upscias )

Source 1 : http://www.columbia.edu/itc/ealac/landesman/summer_public_html/week1/maps/ashokamap.html

Source 2 : https://www.worldhistory.org/Edicts_of_Ashoka/


52. Consider the following pairs:

How many pairs given above are correctly matched?

  1. Only one pair
  2. Only two pairs
  3. Only three pairs
  4. All four pairs

Sol. Ans.(b). Nannuka (c.831−845 CE) was the founder king of the Chandellas.

Jayashakti and Vijayashakti (c. 865−885 CE) Vakpati’s sons Jayashakti (Jeja) and Vijayashakti (Vija) consolidated the Chandella power. According to a Mahoba inscription, the Chandella territory was named “Jejakabhukti” after Jayashakti. So, Jayashakti also belonged to Chandela Dynasty.

Nagabhatta II (c. 800–833 CE): Nagabhatta II was succeeded by his son Ramabhadra, who ruled briefly, and was succeeded by his son Mihira Bhoja. Nagabhatta II belongs to Pratihara Dynasty.

Bhoja I / Mihir Bhoja (c.836−885 CE): Grandson of Nagabhatta II, who had a long reign of over 46 years and proved to be the most successful and popular ruler of Pratiharas. So pair 2 and 4 are not correct.

You were expected to remember the names of all major emperors in major historical dynasties of India. A tough one.

( This question was covered in PT’s Powerpack Course – enquire and enroll here - https://bit.ly/upscias )


53. Which one of the following statements about Sangam literature in ancient South India is correct?

  1. Sangam poems are devoid of any reference to material culture.
  2. The social classification of Varna was known to Sangam poets.
  3. Sangam poems have no reference to warrior ethic
  4. Sangam literature refers to magical forces as irrational.

Sol. Ans.(b). The social classification of Varna was known to the Sangam-era poets. There is mention of Arashar (King) , Vaishiyar (traders) & Velalar (farmers). Brahmins are also mentioned. However, the “4 fold varna classification” had little application to ancient Tamil Society. More relevant basis of classification was Kuti which were clan-based descent groups. Although associated with lineage and hereditary occupations, there were no real restrictions on inter dining and social interactions among Kuti Groups.


54. "Yogavasistha" was translated into Persian by Nizamuddin Panipati during the reign of:

  1. Akbar
  2. Humayun
  3. Shahjahan
  4. Aurangzeb

Sol. Ans.(a). During the Mughal Dynasty the text was translated into Persian several times, as ordered by Akbar, Jahangir and Darah Shikoh. One of these translations was undertaken by Nizam al-Din Panipati in the late sixteenth century AD. A very precise question, requiring specific knowledge.


55. The world's second tallest statue in sitting pose of Ramanuja was inaugurated by the Prime Minister of India at Hyderabad recently. Which one of the following statements correctly represents the teachings of Ramanuja?

  1. The best means of salvation was devotion.
  2. Vedas are eternal, self-existent and wholly authoritative.
  3. Logical arguments were essential means for the highest bliss.
  4. Salvation was to be obtained through meditation.

Sol. Ans.(a). Ramanujacharya was a Hindu theologian, philosopher, and one of the most important supporter of the Sri Vaishnavism tradition within Hinduism. Ramanujacharya's philosophy became known as vishishtadwaita or qualified non-dualism. According to him, devotion and surrender are important for attaining salvation. An easy way to solve this question was to recall that Ramanuja was a Bhakti-era saint. If that is so, then options (b), (c) and (d) are automatically ruled out.

Source : https://www.britannica.com/biography/Ramanuja


56. The Prime Minister recently inaugurated the new Circuit House near Somnath Temple at Veraval. Which of the following statements are correct regarding Somnath Temple?

  1. Somnath Temple is one of the Jyotirlinga shrines.
  2. A description of Somnath Temple was given by Al-Biruni.
  3. Pran Pratishtha of Somnath Temple (installation of the present day temple) was done by President S. Radhakrishnan.

Select the correct answer using the code given below:

  1. 1 and 2 only
  2. 2 and 3 only
  3. 1 and 3 only
  4. 1, 2 and 3

Sol. Ans.(a). A Jyotirlinga or Jyotirlingam, is a devotional representation of the Hindu god Shiva. The word is a Sanskrit compound of jyotis 'radiance' and linga. The Shiva Purana mentions 64 original Jyotirlinga shrines in India, 12 of which are most sacred (the Maha Jyotirlingam) (The Great Jyotirlingas). Somnath is one of them. So, 1 is correct.

The medieval era polymath Al Biruni described Temple's destruction: “In January 1026, Somnath Lingam was smashed, after killing many devotees, and the loot amounted to a huge sum”. So, statement 2 is also correct.

The Pran-pratishtha (installation) of present day Somnath Temple was done by then President of India Dr. Rajendra Prasad. So, 3 is not correct.

Source : https://en.wikipedia.org/wiki/Jyotirlinga


57. Which one of the following statements best describes the role of B cells and T cells in the human body?

  1. They protect the body from environmental allergens.
  2. They alleviate the body’s pain and inflammation.
  3. They act as immunosuppressants in the body.
  4. They protect the body from the diseases caused by pathogens.

Sol. Ans.(d). The B cells create antibodies. B lymphocytes, also called B cells, create a type of protein called an antibody. The T cells can wipe out infected or cancerous cells. They also direct the immune response by helping B lymphocytes to eliminate invading pathogens. This is a question from Biology (general science), or Science and Technology.

Source :
https://www.mdanderson.org/cancerwise/t-cells--b-cells-and-the-immune-system.h00-159465579.html#:~:text=They%20also%20direct%20the%20immune,as%20toxins%2C%20to%20neutralize%20them


58. Consider the following statements:

  1. Other than those made by humans, nanoparticles do not exist in nature.
  2. Nanoparticles of some metallic oxides are used in the manufacture of some cosmetics.
  3. Nanoparticles of some commercial products which enter the environment are unsafe for humans.

Which of the statements given above is/are correct?

  1. 1 only
  2. 3 only
  3. 1 and 2
  4. 2 and 3

Sol. Ans.(d). Statement 1 is not correct. Naturally occurring nanoparticles can be found in volcanic ash, ocean spray, fine sand and dust, and even biological matter (e.g. viruses). So, (a) and (c) are wrong. Now, statement 2 is correct. Applications of inorganic nanoparticles in the cosmetic industry encompass diverse domains, including lip, nail, hair and skin care, and have seen an upsurge in the last few decades. So only option (d) is possible.

( This question was covered in PT’s Powerpack Course – enquire and enroll here - https://bit.ly/upscias )

Source 1 :
https://www.ncbi.nlm.nih.gov/pmc/articles/PMC8417693/

Source 2 :
https://sustainable-nano.com/2013/03/25/nanoparticles-are-all-around-us/#:~:text=Naturally%20occurring%20nanoparticles%20can%20be,than%20their%20naturally%20occurring%20counterparts


59. Consider the following statements:

DNA Barcoding can be a tool to:

  1. assess the age of a plant or animal.
  2. distinguish among species that look alike.
  3. identify undesirable animal or plant materials in processed foods.

Which of the statements given above is/are correct?

  1. 1 only
  2. 3 only
  3. 1 and 2
  4. 2 and 3

Sol. Ans.(d). DNA barcoding is a method of species identification using a short section of DNA from a specific gene or genes. DNA barcoding is a widely used molecular-based system, which can identify biological specimens, and is used for the identification of both raw materials and processed food. So, 2 and 3 are correct, and only possible option is (d).

Source 1 : http://citeseerx.ist.psu.edu/viewdoc/download?doi=10.1.1.717.1235&rep=rep1&type=pdf

Source 2 : https://en.wikipedia.org/wiki/DNA_barcoding


60. Consider the following:

  1. Carbon monoxide
  2. Nitrogen oxide
  3. Ozone
  4. Sulphur dioxide

Excess of which of-the above.-in the environment is/are cause(s) of add rain?

  1. 1, 2 and 3
  2. 2 and 4 only
  3. 4 only
  4. 1, 3 and 4

Sol. Ans.(b). Acid rain is caused by a chemical reaction that begins when compounds like sulfur dioxide and nitrogen oxides are released into the air. These substances can rise very high into the atmosphere, where they mix and react with water, oxygen, and other chemicals to form more acidic pollutants, known as acid rain. So, only 2 and 4 are correct.


61. Consider the following statements:

  1. High clouds primarily reflect solar radiation and cool the surface of the Earth.
  2. Low clouds have a high absorption of infrared radiation emanating from the Earth's surface and thus cause warming effect.

Which of the statements given above is/are correct?

  1. 1 only
  2. 2 only
  3. Both 1 and 2
  4. Neither 1 nor 2

Sol. Ans.(d). Both the statements are wrong, and their reverse would be right.

High, thin clouds primarily transmit incoming solar radiation; at the same time, they trap some of the outgoing infrared radiation emitted by the Earth and radiate it back downward, thereby warming the surface of the Earth. Low, thick clouds primarily reflect solar radiation and cool the surface of the Earth. So, both statements are not correct.

Source : https://earthobservatory.nasa.gov/features/Clouds#:~:text=Low%2C%20thick%20clouds%20primarily%20reflect,the%20surface%20of%20the%20Earth


62. Consider the following statements:

  1. Bidibidi is a large refugee settlement in north-western Kenya.
  2. Some people who fled from South Sudan civil war live in Bidibidi.
  3. Some people who fled from civil war in Somalia live in Dadaab refugee complex in Kenya.

Which of the statements given above is/are correct?

  1. 1 and 2
  2. 2 only
  3. 2 and 3
  4. 3 only

Sol. Ans.(c). Bidibidi Refugee Settlement is a refugee camp in northwestern Uganda (and not Kenya, as given in question). With over 270,000 South Sudanese refugees fleeing the ongoing civil war, as of early 2017 it was the largest refugee settlement in the world. So, 1 is not correct and 2 is correct.

The Dadaab refugee complex has a population of 2,18,873 registered refugees and asylum seekers as at the end of July 2020. Dadaab refugee complex consists of three camps. The first camp was established in 1991, when refugees fleeing the civil war in Somalia started to cross the border into Kenya. A second large influx occurred in 2011, when some 130,000 refugees arrived, fleeing drought and famine in southern Somalia.


63. Consider the following countries:

  1. Armenia
  2. Azerbaijan
  3. Croatia
  4. Romania
  5. Uzbekistan

Which of the above are members of the Organization of Turkic States?

  1. 1, 2 and 4
  2. 1 and 3
  3. 2 and 5
  4. 3, 4 and 5

Sol. Ans.(c). The Organization of Turkic States, formerly called the Turkic Council or the Cooperation Council of Turkic Speaking States, is an international organization comprising prominent independent Turkic countries: Azerbaijan, Kazakhstan, Kyrgyzstan, Turkey and Uzbekistan. Our of the given countries 2 (Azerbaijan) and 5 (Uzbekistan) is correct. (If you cannot remember all member names, then in this particular question, try thinking who all were part of the Turkic groups that came into India starting the 11th century – clearly, 1, 2 and 4 were not).


64. Consider the following statements:

  1. Gujarat has the largest solar park in India.
  2. Kerala has a fully solar powered International Airport.
  3. Goa has the largest floating solar photovoltaic project in India.

Which of the statements given above is/are correct?

  1. 1 and 2
  2. 2 only
  3. 1 and 3
  4. 3 only

Sol. Ans.(b). The Bhadla Solar Park, Jodhpur, Rajasthan is the largest solar power park not only India but in the world. It spans 14,000 acres. There are over 10 million solar panels at the park, which contribute to an operational capacity of 2245MW. So, 1 is not correct. So (a) and (c) are ruled out. The firm Bharat Heavy Electricals Limited (BHEL) has successfully commissioned India’s largest Floating Solar PV plant. Located at NTPC Simhadri in Andhra Pradesh, the 25 MW floating SPV project covers an area of 100 acres. So, 3 is not correct. Cochin International Airport Ltd (CIAL) owns the world’s first airport fully powered by solar energy.

Only possible option is (b).

Source : https://www.thehindu.com/sci-tech/energy-and-environment/worlds-largest-solar-park-in-bhadla-india/article37462665.ece

Source :
https://www.livemint.com/industry/energy/indias-largest-floating-solar-power-plant-commissioned-in-andhra-pradesh-11631774123853.html

Source : https://www.indiatimes.com/news/india/kochi-airport-more-solar-power-with-floating-solar-plant-532356.html


65. With reference to the United Nations Convention on the Law of Sea, consider the following statements:

  1. A coastal state has the right to establish the breadth of its territorial sea up to a limit not exceeding 12 nautical miles, measured from baseline determined in accordance with the convention.
  2. Ships of all states, whether coastal or land-locked, enjoy the right of innocent passage through the territorial sea.
  3. The Exclusive Economic Zone shall not extend beyond 200 nautical miles from the baseline from which the breadth of the territorial sea is measured.

Which of the statements given above are correct?

  1. 1 and 2 only
  2. 2 and 3 only
  3. 1 and 3 only
  4. 1, 2 and 3

Sol. Ans.(d). ) Every State has the right to establish the breadth of its territorial sea up to a limit not exceeding 12 nautical miles, measured from baselines determined in accordance with this Convention. So, 1 is correct. Subject to this Convention, ships of all States, whether coastal or land-locked, enjoy the right of innocent passage through the territorial sea. So, 2 is also correct. An “exclusive economic zone,” or “EEZ” is an area of the ocean, generally extending 200 nautical miles (230 miles) beyond a nation’s territorial sea, within which a coastal nation has jurisdiction over both living and nonliving resources. So, 3 is also correct.

( This question was covered in PT’s Powerpack Course – enquire and enroll here - https://bit.ly/upscias )

Source : https://www.un.org/depts/los/convention_agreements/texts/unclos/part2.htm


66. Which one of the following statements best reflects the issue with Senkaku Islands, sometimes mentioned in the news?

  1. It is generally believed that they are artificial islands made by a country around South China Sea.
  2. China and Japan engage in maritime disputes over these islands in East China Sea.
  3. A permanent American military base has been set up there to help Taiwan to increase its defence capabilities.
  4. Though International Court of Justice declared them as no man's land, some South-East Asian countries claim them.

Sol. Ans.(b). The Senkaku Islands dispute, or Diaoyu Islands dispute, is a territorial dispute over a group of uninhabited islands known as the Senkaku Islands in Japan, the Diaoyu Islands in the People's Republic of China (PRC), and Tiaoyutai Islands in the Republic of China (ROC or Taiwan). This is a pretty old question being asked after many years.

( This question was covered in PT’s Powerpack Course – enquire and enroll here - https://bit.ly/upscias )


67. Consider the following pairs:

How many pairs given above are correctly matched?

  1. Only one pair
  2. Only two pairs
  3. Only three pairs
  4. All four pairs

Sol. Ans.(c). The Chinese People's Liberation Army Support Base in Djibouti is a military base operated by the Chinese People's Liberation Army Navy (PLAN), located in Djibouti in the Horn of Africa. Other three are correctly matched. In this variety of questions, you need to know about all four things asked else the question cannot be properly solved.


68. Consider the following pairs:

How many pairs given above are correctly matched?

  1. Only one pair
  2. Only two pairs
  3. Only three pairs
  4. All four pairs

Sol. Ans.(b). ) Anatolia, also known as Asia Minor, is a large peninsula in Western Asia and the westernmost protrusion of the Asian continent. It constitutes the major part of modern-day Turkey.

Amharas are a Semitic-speaking ethnic group which is indigenous to Ethiopia, traditionally inhabiting parts of the northwest Highlands of Ethiopia, particularly inhabiting the Amhara Region.

Cabo Delgado is the northernmost province of Mozambique. (not Spain)

Catalonia is an autonomous community of Spain.

So, only two pairs are correctly matched.


69. With reference to Indian laws about wildlife protection, consider the following statements:

  1. Wild animals are the sole property of the government.
  2. When a wild animal is declared protected, such animal is entitled for equal protection whether it is found in protected areas or outside.
  3. Apprehension of a protected wild animal becoming a danger to human life is sufficient ground for its capture or killing.

Which of the statements given above is/are correct?

  1. 1 and 2
  2. 2 only
  3. 1 and 3
  4. 3 only

Sol. Ans.(a). In a significant verdict, the Bombay High Court has ruled that wild animals including tiger should be treated as “government property for all purposes”. So, 1 is correct.

THE WILD LIFE (PROTECTION) ACT, 1972 - Prohibition of hunting.—No person shall hunt any wild animal specified in Schedules I, II, III and IV except as provided under section 11 and section 12.

Hunting of wild animals to be permitted in certain cases -

  1. Notwithstanding anything contained in any other law for the time being in force and subject to the provisions of Chapter IV -
  1. the Chief Wild Life Warden may, if he is satisfied that any wild animal specified in Schedule I has become dangerous to human life or is so disabled or diseased as to be beyond recovery, by order in writing and stating the reasons therefor, permit any person to hunt such animal or cause such animal to be hunted: 1[Provided that no wild animal shall be ordered to be killed unless the Chief Wild Life Warden is satisfied that such animal cannot be captured, tranquilized or translocated: Provided further that no such captured animal shall be kept in captivity unless the Chief Wild Life Warden is satisfied that such animal cannot be rehabilitated in the wild and the reasons for the same are recorded in writing.

Explanation - For the purposes of clause (a), the process of capture or translocation, as the case may be, of such animal shall be made in such manner as to cause minimum trauma to the said animal.]

So, 3 is not correct.

Source : https://zeenews.india.com/news/eco-news/treat-wild-animals-as-govt-property-for-all-purposes_769840.html


70. Certain species of which one of the following organisms are well known as cultivators of fungi?

  1. Ant
  2. Cockroach
  3. Crab
  4. Spider

Sol. Ans.(a). Ants are the world’s best fungus farmers. This is an amazing example of mutualism or symbiosis.


71. Consider the following statements:

  1. Tight monetary policy of US Federal Reserve could lead to capital flight.
  2. Capital flight may increase the interest cost of firms with existing External Commercial Borrowings (ECBs).
  3. Devaluation of domestic currency decreases the currency risk associated with ECBs.

Which of the statements given above are correct?

  1. 1 and 2 only
  2. 2 and 3 only
  3. 1 and 3 only
  4. 1, 2 and 3

Sol. Ans.(a). Statements 1 and 2 are right, but 3 is not. Tight monetary policy of the US Fed will mean higher interest rates, that will mean capital flight out of India, and consequent effect.

A depreciation of the domestic currency will lead to an increase in the value of foreign currency debts in domestic currency terms. So, a significant depreciation of the local currency would translate into an increase in the local-currency value of outstanding debt. As a consequence, this would lead to a deterioration of the debt-servicing capacity of domestic borrowers. Note that “External Commercial Borrowing (ECB)” is a type of loan in foreign currencies, made by non-resident lenders.

( This question was covered in PT’s Powerpack Course – enquire and enroll here - https://bit.ly/upscias )


72. Consider the following States:

  1. Andhra Pradesh
  2. Kerala
  3. Himachal Pradesh
  4. Tripura

How many of the above are generally known as tea-producing States?

  1. Only one State
  2. Only two States
  3. Only three States
  4. All four States

Sol. Ans.(a). Assam, West Bengal, Tamil Nadu and Kerala accounts for 98% of total tea production in India. So, correct answer is (a). [there is some confusion in this question due to the wording “generally”, hence other answers are also possible]


73. Consider the following statements:

  1. In India, credit rating agencies are regulated by Reserve Bank of India.
  2. The rating agency popularly known as ICRA is a public limited company.
  3. Brickwork Ratings is an Indian credit rating agency.

Which of the statements given above are correct?

  1. 1 and 2 only
  2. 2 and 3 only
  3. 1 and 3 only
  4. 1, 2 and 3

Sol. Ans.(b). In India, the various credit rating agencies are regulated by SEBI (Securities and Exchanges Board of India) and not the RBI (Reserve Bank of India). So, 1 is not correct, and only possible option is (b), as three options are ruled out.

( This question was covered in PT’s Powerpack Course – enquire and enroll here - https://bit.ly/upscias )


74. With reference to the ‘Banks Board Bureau (BBB)', which of the following statements are correct?

  1. The Governor of RBI is the Chairman of BBB.
  2. BBB recommends for the selection of heads for Public Sector Banks.
  3. BBB helps the Public Sector Banks in developing strategies and capital raising plans.

Select the correct answer using the code given below:

  1. 1 and 2 only
  2. 2 and 3 only
  3. 1 and 3 only
  4. 1, 2 and 3

Sol. Ans.(b). The Governor of RBI is not the chairman of the Bank Board Bureau (BBB). So, 1 is not correct, and the only possible option is (b)

( This question was covered in PT’s Powerpack Course – enquire and enroll here - https://bit.ly/upscias )


75. With reference to Convertible Bonds, consider the following statements:

  1. As there is an option to exchange the bond for equity. Convertible Bonds pay a lower rate of interest.
  2. The option to convert to equity affords the bondholder a degree of indexation to rising consumer prices.

Which of the statements given above is/are correct?

  1. 1 only
  2. 2 only
  3. Both 1 and 2
  4. Neither 1 nor 2

Sol. Ans.(c). Convertible bonds tend to offer a lower coupon rate or rate of return in exchange for the value of the option to convert the bond into common stock. Companies benefit since they can issue debt at lower interest rates than with traditional bond offerings but not all companies offer convertible bonds.

The option to convert to equity affords the bondholder a degree of indexation to rising consumer prices. So, both are correct.


76. Consider the following:

  1. Asian Infrastructure Investment Bank
  2. Missile Technology Control Regime
  3. Shanghai Cooperation Organisation

India is a member of which of the above?

  1. 1 and 2 only
  2. 3 only
  3. 2 and 3 only
  4. 1, 2 and 3

Sol. Ans.(d). India is a member of Asian Infrastructure Investment Bank (AIIB) and has been the largest borrower of AIIB lending since the time the Bank started its operations.

The SCO (Shanghai Cooperation Organisation) currently comprises eight Member States (China, India, Kazakhstan, Kyrgyzstan, Russia, Pakistan, Tajikistan and Uzbekistan), four Observer States interested in acceding to full membership (Afghanistan, Belarus, Iran, and Mongolia) and six “Dialogue Partners”. So 1 and 3 are correct. Only possible option is (d).


77. Consider the following statements:

  1. Vietnam has been one of the fastest growing economies in the world in the recent years.
  2. Vietnam is led by a multi-party political system.
  3. Vietnam's economic growth is linked to its integration with global supply chains and focus on exports.
  4. For a long time Vietnam's low labour costs and stable exchange rates have attracted global manufacturers.
  5. Vietnam has the most productive e-service sector in the Indo-Pacific region.

Which of the statements given above are correct?

  1. 2 and 4
  2. 3 and 5
  3. 1, 3 and 4
  4. 1 and 2

Sol. Ans.(c). A recent GDP growth statistics report states that Vietnam is the fastest growing world economy with a growth rate of 7.31% in 2019. It has surpassed the growth rate of other Asian economies including India and China, which stand at 4.5% and 6% CAGR respectively. So, 1 is correct.

Vietnam is unitary, socialist, communist, one-party state. So, 2 is not correct. Only possible option is (c).

Source :
https://locus.sh/resources/bulletin/vietnams-economic-growth/#:~:text=A%20recent%20GDP%20growth%20statistics,%25%20and%206%25%20CAGR%20respectively


78. In India, which one of the following is responsible for maintaining price stability by controlling inflation?

  1. Department of Consumer Affairs
  2. Expenditure Management Commission
  3. Financial Stability and Development Council
  4. Reserve Bank of India

Sol. Ans.(d). ) A key role of RBI and other central bank is to conduct monetary policy to achieve price stability (low and stable inflation) and to help manage economic fluctuations. This question was a sitter.

( This question was covered in PT’s Powerpack Course – enquire and enroll here - https://bit.ly/upscias )


79. With reference to Non-Fungible Tokens (NFTs), consider the following statements:

  1. They enable the digital representation of physical assets.
  2. They are unique cryptographic tokens that exist on a block chain.
  3. They can be traded or exchanged at equivalency and therefore can be used as a medium of commercial transactions.

Which of the statements given above are correct?

  1. 1 and 2 only
  2. 2 and 3 only
  3. 1 and 3 only
  4. 1, 2 and 3

Sol. Ans.(a). ) A non-fungible token (NFT) is a unique digital asset that represents ownership of real-world items like art, video clips, music, and more. NFTs use the same blockchain technology that powers cryptocurrencies, but they’re not a currency. It cannot be used as a medium for commercial transactions.

( This question was covered in PT’s Powerpack Course – enquire and enroll here - https://bit.ly/upscias )


80. Consider the following pairs:

How many pairs given above are not correctly matched?

  1. Only one pair
  2. Only two pairs
  3. Only three pairs
  4. All four pairs

Sol. Ans.(c). Ghatprabha- Karnataka, Gandhi Sagar- Madhya Pradesh, Indira Sagar- Madhya Pradesh, Maithon- Jharkhand. So THREE pairs are NOT correct. This is a tough question because elimination of options is ruled out. You must know all relations to be able to solve it properly.


81. In the Government of India Act 1919, the functions of Provincial Government were divided into "Reserved" and "Transferred" subjects. Which of the following were treated as "Reserved" subjects?

  1. Administration of Justice
  2. Local Self-Government
  3. Land Revenue
  4. Police

Select the correct answer using the code given below:

  1. 1, 2 and 3
  2. 2, 3 and 4
  3. 1, 3 and 4
  4. 1, 2 and 4

Sol. Ans.(c). ‘Local Self Government’ was included in the ‘Transferred List’. So, only possible option is (c). A small trick to solve this is to look at all the subjects mentioned – Justice administration, Local self-govt., Land revenue and Police. Other than local self government, the other three are clearly strategic in nature that the British wouldn’t want to delegate to local political units.

( This question was covered in PT’s Powerpack Course – enquire and enroll here - https://bit.ly/upscias )

Source :
https://en.wikipedia.org/wiki/Government_of_India_Act_1919#:~:text=In%20each%20such%20province%2C%20control,The%20Provincial%20Councils%20were%20enlarged


82. In medieval India, the term "Fanam" referred to:

  1. Clothing
  2. Coins
  3. Ornaments
  4. Weapons

Sol. Ans.(b). Fanam was a currency issued in Madras Presidency, now part of Tamil Nadu, India and Travancore State, now part of Kerala, India. It was also in the Ganga Dynasty period. https://en.wikipedia.org/wiki/Fanam

( This question was covered in PT’s Powerpack Course – enquire and enroll here - https://bit.ly/upscias )


83. Consider the following freedom fighters:

  1. Barindra Kumar Ghosh
  2. Jogesh Chandra Chatterjee
  3. Rash Behari Bose

Who of the above was/were actively associated with the Ghadar Party?

  1. 1 and 2
  2. 2 only
  3. 1 and 3
  4. 3 only

Sol. Ans.(d). Rash Behari Bose (25 May 1886 – 21 January 1945) was an Indian revolutionary leader against the British Raj. He was one of the key organisers of the Ghadar Mutiny and founded the First Indian National Army during World War 2 based on millitarization policy of Vinayak Damodar Savarkar. Jogesh Chandra Chatterjee was associated with Hindustan Republican Association (HRA).

( This question was covered in PT’s Powerpack Course – enquire and enroll here - https://bit.ly/upscias )

Source : https://en.wikipedia.org/wiki/Jogesh_Chandra_Chatterjee

Source : https://en.wikipedia.org/wiki/Barindra_Kumar_Ghosh

Source : https://en.wikipedia.org/wiki/Rash_Behari_Bose


84. With reference to the proposals of Cripps Mission, consider the following statements:

  1. The Constituent Assembly would have members nominated by the Provincial Assemblies as well as the Princely States.
  2. Any Province, which is not prepared to accept the new Constitution would have the right to sign a separate agreement with Britain regarding its future status.

Which of the statements given above is/are correct?

  1. 1 only
  2. 2 only
  3. Both 1 and 2
  4. Neither 1 nor 2

Sol. Ans.(c). Proposals of Cripps Mission were -

  1. Setting up of an Indian dominion. This dominion would have the freedom to remain with the British Commonwealth or to secede from it. It would also be at liberty to take part in international organisations.
  2. A Constituent Assembly would be formed to frame a new constitution for the country. This Assembly would have members elected by the provincial assemblies and also nominated by the princes.
  3. Any province unwilling to join the Indian dominion could form a separate union and have a separate constitution.
  4. The transfer of power and the rights of minorities would be safeguarded by negotiations between the Constituent Assembly and the British government.
  5. In the meantime, until this new constitution came into force, India’s defence would be controlled by the British and the powers of the Governor-General would remain unaltered.
  6. Any province unwilling to join the Indian dominion could form a separate union and have a separate constitution.
  7. So, both the statements are correct.

    ( This question was covered in PT’s Powerpack Course – enquire and enroll here - https://bit.ly/upscias )


85. With reference to Indian history, consider the following texts:

  1. Nettipakarana
  2. Parishishtaparvan
  3. Avadanashataka
  4. Trishashtilakshana Mahapurana

Which of the above are Jaina texts?

  1. 1, 2 and 3
  2. 2 and 4 only
  3. 1, 3 and 4
  4. 2, 3 and 4

Sol. Ans.(b). The Avadānaśataka or “Century of Noble Deeds (Avadāna)” is an anthology in Sanskrit of one hundred Buddhist legends, approximately dating to the same time as the Ashokavadana. So, 3 is not Jaina Text, and hence three options are ruled out. Only possible option is (b). A good question from ancient and medieval India.

Source : https://en.wikipedia.org/wiki/Avadanasataka


86. With reference to Indian history, consider the following pairs:

How many pairs given above are correctly matched?

  1. None of the pairs
  2. Only one pair
  3. Only two pairs
  4. All three pairs

Sol. Ans.(c). Āryadeva (3rd century CE) was a Mahayana Buddhist monk, a disciple of Nagarjuna and a Madhyamaka philosopher. Most sources agree that he was from "Siṃhala", which some scholars identify with Sri Lanka.

Dignāga (c. 480 – c. 540 CE) was an Indian Buddhist scholar and one of the Buddhist founders of Indian logic.

Sri Ranganathamuni, popularly known as Sriman Nathamuni (823 CE–951 CE), was a Vaishnava theologian who collected and compiled the Nalayira Divya Prabandham.

Only two pairs are matched.


87. With reference to Indian history, consider the following statements:

  1. The first Mongol invasion of India happened during the reign of Jalal-ud-din Khalji.
  2. During the reign of Ala-ud-din Khalji, one Mongol assault marched up to Delhi and besieged the city.
  3. Muhammad-bin-Tughlaq temporarily lost portions of north-west of his kingdom to Mongols.

Which of the statements given above is/are correct?

  1. 1 and 2
  2. 2 only
  3. 1 and 3
  4. 3 only

Sol. Ans.(b and d). The first (possible) Mongol invasion took place in the reign of Iltutmish, though Ghenghis Khan returned from the Indus. But later, there were other attacks by Mongols, prior to Jalal-ud-din’s reign. Mongol assault did besiege the city of Delhi during the reign of Ala-ud-din Khilji. So, 2 is correct. Statement 3 is also correct. Hence, no option is correct.


88. With reference to Indian history, who of the following were known as "Kulah-Daran"?

  1. Arab merchants
  2. Qalandars
  3. Persian calligraphists
  4. Sayyids

Sol. Ans.(d). The Sayyids claimed their descent from the Prophet through his daughter Fatima. They commanded special respect in the Muslim society. Even Timurlane protected the life of Sayyids during his invasion in India, although his policy was one of general slaughter. The Sayyids put on a pointed cap (kulah) and they were known as ‘Kulah Daran’ during Delhi sultanate.

Source :
https://books.google.co.in/books?id=nMWSQuf4oSIC&printsec=frontcover&source=gbs_ge_summary_r&cad=0#v=onepage&q=Kulah%20Daran&f=false


89. With reference to Indian history, consider the following statements:

  1. The Dutch established their factories/warehouses on the east coast on lands granted to them by Gajapati rulers.
  2. Alfonso de Albuquerque captured Goa from the Bijapur Sultanate.
  3. The English East India Company established a factory at Madras on a plot of land leased from a representative of the Vijayanagara empire.

Which of the statements given above are correct?

  1. 1 and 2 only
  2. 2 and 3 only
  3. 1 and 3 only
  4. 1, 2 and 3

Sol. Ans.(b). The first Dutch colony was established in 1602 whereas Gajapati empire ended in 1541. The last ruler was Kakharua Deva. So, 1 cannot be correct. So three options are ruled out instantly.

Alfanso de Albuquerque captured Goa from the Bijapur sultanate king Adil Shahis with the help of Vijaynagara Empire. So, 2 is correct.

The English East India Company established a factory in Madras in 1639 on land leased from representatives of Vijayanagara Empire called the Nayakas. So, 3 is correct.

Source : https://www.thehindu.com/features/kids/how-madras-came-to-be-376-years-ago/article7561549.ece


90. According to Kautilya's Arthashastra, which of the following are correct?

  1. A person could be a slave as a result of a judicial punishment.
  2. If a female slave bore her master a son, she was legally free.
  3. If a son born to a female slave was fathered by her master, the son was entitled to the legal status of the master's son-

Which of the statements given above are correct?

  1. 1 and 2 only
  2. 2 and 3 only
  3. 1 and 3 only
  4. 1, 2 and 3

Sol. Ans.(d). The Arthashastra states that a man could be a slave either by birth, by voluntarily selling himself, by being captured in war, or as a result of a judicial punishment. Slavery was a recognized institution and the legal relationship between master and slave was clearly defined e.g. if a female slave bore her master a son, not only was she legally free but the child was entitled to legal status of the master’s son. So, all three statements are correct.

Source : https://ncjindalps.com/pdf/HUMANITIES/The%20Kautilya%20Arthashastra%20-%20Chanakya.pdf

Source : http://indiansaga.com/others/slavery.html


91. In India, which one of the following compiles information on industrial disputes, closures, retrenchments and lay-offs in factories employing workers?

  1. Central Statistics Office
  2. Department for Promotion of Industry and Internal Trade
  3. Labour Bureau
  4. National Technical Manpower Information System

Sol. Ans.(c). The Labour Bureau compiles information on industrial disputes, closures, retrenchments and lay-offs.

Source : http://labourbureaunew.gov.in/UserContent/Statistics_ID_Layoffs_2011.pdf


92. In India, what is the role of the Coal Controller's Organization (CCO)?

  1. CCO is the major source of Coal Statistics in Government of India.
  2. It monitors progress of development of Captive Coal/Lignite blocks.
  3. It hears any objection to the Governments notification relating to acquisition of coal-bearing areas.
  4. It ensures that coal mining companies deliver the coal to end users in the prescribed time.

Select the correct answer using the code given below:

  1. 1, 2 and 3
  2. 3 and 4 only
  3. 1 and 2 only
  4. 1, 2 and 4

Sol. Ans.(a). Under the Collection of Statistics Act, 2008 - Coal Controller has been made the statistical authority with respect to coal and lignite statistics. Entrusted the responsibility of carrying out Annual Coal & Lignite survey and publishing of Provisional Coal Statistics and Coal Directory of India. Submission of monthly coal data to different ministries of central and state Govt., national and international organization. Collection of Statistics relating to coal washeries.

Under Coal Bearing Area (Acquisition and Development) Act, 1957- Coal Controller is the competent authority under this act to hear any objection to the Central Government’s Notification relating to acquisition of coal bearing land and to furnish his reports to Central Govt. Under the Coking Coal Mines (Nationalisation) act, 1972, the Non-coking Coal Mines (Nationalisation) Act, 1973 and the Coal Mines (Special Provision) Act, 2015. Coal Controller also functions as the Commissioner of Payment to settle the claim cases of colliery owners of pre-nationalisation period under the above acts.

Source : http://www.coalcontroller.gov.in/pages/display/5-functionsresponsibilities


93. If a particular area is brought under the Fifth Schedule of the Constitution of India, which one of the following statements best reflects the consequence of it?

  1. This would prevent the transfer of land of tribal people to non-tribal people.
  2. This would create a local self-governing body in that area.
  3. This would convert that area into a Union Territory.
  4. The State having such areas would be declared a Special Category State.

Sol. Ans.(a). The Fifth Schedule of the Constitution deals with the administration and control of Scheduled Areas as well as of Scheduled Tribes residing in any State other than the States of Assam, Meghalaya, Tripura and Mizoram. This prohibits or restricts the transfer of land by or among members of the Scheduled tribes in such area.

( This question was covered in PT’s Powerpack Course – enquire and enroll here - https://bit.ly/upscias )

Source :
https://vikaspedia.in/social-welfare/scheduled-tribes-welfare/fifth-schedule-areas#:~:text=The%20Fifth%20Schedule%20of%20the%20Constitution%20deals%20with%20the%20administration,%2C%20Meghalaya%2C%20Tripura%20and%20Mizoram


94. Consider the following statements:

  1. The India Sanitation Coalition is a platform to promote sustainable sanitation and is funded by the Government of India and the World Health Organization.
  2. The National Institute of Urban Affairs is an apex body of the Ministry of Housing and Urban Affairs in Government of India and provides innovative solutions to address the challenges of Urban India.

Which of the statements given above is/are correct?

  1. 1 only
  2. 2 only
  3. Both 1 and 2
  4. Neither 1 nor 2

Sol. Ans.(b). The India Sanitation Coalition (ISC), launched in June 2015, at Federation of Indian Chamber Commerce and Industry (FICCI), enables and supports safe and sustainable sanitation by bringing multiple organizations on a common platform through a range of catalytic actions. These include supporting the unlocking of WASH financing with focus on the private sector, forging partnerships with allied organizations for leading the discourse on sustainable sanitation; convening, curating and disseminating best practices in the sanitation advocacy — space and providing inputs into the policy aspects of sanitation through participation at allied forums. So it is not funded by either Govt. of India or by WHO. Statement 1 is not correct. The NIUA was appointed as an apex body to support and guide the Government of India in its urban development plans. Since then, it has worked closely with the Ministry of Housing and Urban Affairs. Statement 2 is correct. This is a question from “government schemes”.

Source : http://urbanrivers.niua.org/node/46

Source : https://www.indiasanitationcoalition.org/who-we-are.html


95. Which one of the following has been constituted under the Environment (Protection) Act, 1986?

  1. Central Water Commission
  2. Central Ground Water Board
  3. Central Ground Water Authority
  4. National Water Development Agency

Sol. Ans.(c). The Central Ground Water Authority (CGWA) was constituted under sub-section (3) of Section 3 of the Environment (Protection) Act, 1986 for the purposes of regulation and control of ground water development and management in the country.

( This question was covered in PT’s Powerpack Course – enquire and enroll here - https://bit.ly/upscias )


96. With reference to the ''United Nations Credentials Committee", consider the following statements:

  1. It is a committee set up by the UN Security Council and works under its supervision.
  2. It traditionally meets in March, June and September every year.
  3. It assesses the credentials of al! UN members before submitting a report to the General Assembly for approval.

Which of the statements given above is/are correct?

  1. 3 only
  2. 1 and 3
  3. 2 and 3
  4. 1 and 2

Sol. Ans.(a). The UN Credentials Committee is part of UNGA and not the UN Security Council as given in question. So, 1 is not correct. There is no mention of meeting it at regular intervals. So, 2 is not correct.

Source : https://www.un.org/en/ga/credentials/credentials.shtml


97. Which one of the following statements best describes the 'Polar Code?

  1. It is the international code of safety for ships operating in polar waters.
  2. It is the agreement of the countries around the North Pole regarding the demarcation of their territories in the polar region.
  3. It is a set of norms to be followed by the countries whose scientists undertake research studies in the North Pole and South Pole.
  4. It is a trade and security agreement of the member countries of the Arctic Council.

Sol. Ans.(a). IMO's International Code for Ships Operating in Polar Waters (Polar Code) is mandatory under both the International Convention for the Safety of Life at Sea (SOLAS) and the International Convention for the Prevention of Pollution from Ships (MARPOL). The Polar Code covers the full range of design, construction, equipment, operational, training, search and rescue and environmental protection matters relevant to ships operating in the inhospitable waters surrounding the two poles. IMO’s International Code for Ships Operating in Polar Waters (Polar Code) is mandatory under both the International Convention for the Safety of Life at Sea (SOLAS) and the International Convention for the Prevention of Pollution from Ships (MARPOL).

Source : https://www.imo.org/en/OurWork/Safety/Pages/polar-code.aspx


98. With reference to the United Nations General Assembly, consider the following statements:

  1. The UN General Assembly can grant observer status to the non-member States.
  2. Inter-governmental organisations can seek observer status in the UN General Assembly.
  3. Permanent Observers in the UN General Assembly can maintain missions at the UN headquarters.

Which of the statements given above are correct?

  1. 1 and 2 only
  2. 2 and 3 only
  3. 1 and 3 only
  4. 1, 2 and 3

Sol. Ans.(d). The United Nations General Assembly may grant non-member states, international organisations and other entities Permanent Observer Status. Permanent Observers may participate in the sessions and workings of the General Assembly and maintain missions at the UN Headquarters.

Source : https://ask.un.org/faq/14519


99. With reference to the "Tea Board" in India, consider the following statements:

  1. The Tea Board is a statutory body.
  2. It is a regulatory body attached to the Ministry of Agriculture and Farmers Welfare.
  3. The Tea Board's Head Office is situated in Bengaluru.
  4. The Board has overseas offices at Dubai and Moscow.

Which of the statements given above are correct?

  1. 1 and 3
  2. 2 and 4
  3. 3 and 4
  4. 1 and 4

Sol. Ans.(d). The Tea Board set up under section 4 of the Tea Act 1953 was constituted on 1st April 1954.

Tea Board is functioning as a statutory body of the Central Government under the Ministry of Commerce.

Tea Board’s head office is situated in Kolkata.

Source : https://www.teaboard.gov.in/TEABOARDCSM/NA==


100. Which one of the following best describes the term "green washing"?

  1. Conveying a false impression that a company's products are eco-friendly and environmentally sound
  2. Non-inclusion of ecological/ environmental costs in the Annual Financial Statements of a country
  3. Ignoring the disastrous ecological consequences while undertaking infrastructure development
  4. Making mandatory provisions for environmental costs in a government project/programme

Sol. Ans.(a). Greenwashing is the process of conveying a false impression or providing misleading information about how a company’s products are more environmentally sound. It is considered an unsubstantiated claim to deceive consumers into believing that a company’s products are environmentally friendly.

( This question was covered in PT’s Powerpack Course – enquire and enroll here - https://bit.ly/upscias )

Source : https://www.financialexpress.com/opinion/greenwashing-is-an-elusive-csr-attempt/1332364/





Questions and Detailed Solutions are being continuously updated ... refresh and check. Comment and let us know your experience, answers and solutions too!